Sie sind auf Seite 1von 104

D01

LOGO

CHARTERED INSURANCE INSTITUTE OF NIGERIA


DIPLOMA

OCTOBER 2013 EXAMINATION PAPER

SUBJECT D01
INSURANCE PRACTICE AND REGULATION

Three hours are allowed for this paper

Fill in the information requested on the answer booklet and on form B.

Do not write on the question paper.

The answer booklet and this question paper must be handed in personally
by you to the invigilator before you leave the Examination Hall. Failure to
do this may result in your paper not being marked and you may be
prevented from entering for this examination in future.
D01
CHARTERED INSURANCE INSTITUTE OF NIGERIA

D01 INSURANCE PRACTICE AND REGULATION

INSTRUCTIONS TO CANDIDATES

Three (3) hours are allowed for this paper which contains 20 compulsory
questions and carries a total of 200 marks.

Where a question is split into parts (a), (b) etc, marks for each part are only
shown if they are split unevenly between the parts and you should spend your
time in accordance with the allocation.

Answer each question on a new page. If a question has more than one part,
leave several lines blank after each part.

It is important to show each step in any calculation, even if you have used a
calculator.

You may find it helpful in some places to make rough notes in the answer
booklet, if you do this, you should cross through these notes before you hand in
the booklet.

D01 INSURANCE PRACTICE AND REGULATION


1. Outline the importance of Cover Note and Certificate of Insurance in Motor
Insurance Contract.
(10 marks)
Solution 1
Cover note and certificate of Insurance are issued to motor policyholder in
accordance with the Road Traffic Act, 1988 section 143 which stipulate that any
person who put his vehicle on road should have a valid third party or Act only
policy to take care of his legal liability to third parties property damage or bodily
injury/death.

- Cover note is issued as temporary cover for 15days or 30days in motor


insurance contract, while certificate of insurance is issued for 12 months
from the inception date.

The importance of Cover Note and Certificate of Insurance are:

- Both serve as evidence that the insurance is in force in accordance with the
provision of the law
- Where the policy document is not issued, this holds as basis of the contract
- they show the period of insurance cover
- they show the type of cover granted to the insured
- and show the limitation of use i.e. geographical limitation.

2. Illustrate with example the structure of Nigeria Insurance market.


(10 marks)
Solution 2
The structure of Nigeria Insurance market comprises

a) The Buyers (insured) includes public sector (i.e. Government/parastatals),


commerce, industry, financial institutions, manufacturing, multinational,
Telecommunication etc.
b) The intermediaries are mainly agents and Insurance Brokers.

c) The insurers: life companies, general companies, composite companies,


and Reinsurers and Actuarist.
d) Insurance Superintendent, and Loss Adjusters.

3. Outline the main factors that are considered in calculating premium in Life
and General insurance contract. (10 marks)
Solution 3
The main factors in calculating premium in both life and general insurance are:
- provision for expected claims (IBNR) i.e. Incurred But Not Reported
- Provision for operational expenses or overhead expenses e.g. staff salary,
commission, Management expenses etc.
- Provision for Profit margin
- Provision for General inflation
- Provision for interest Rate and Exchange rates
- Mortality rates/age next birthday
- Competitiveness of the Market.

4. Compare the claim procedure in general insurance and life Insurance


Contract. (10 marks)

Solution 4:
Claim Procedure/Processes in Life Assurance are as follows:
Maturity Claim: The Insurer is obliged to notify the insured two or three months
in advance before the maturity date of the life Assurance policy.

Death Claim: Notification to Insurer by the deceased family or executors


- Proof of death is also required by the Insurer e.g. death and burial
certificate.
- Assignment of proceeds of death benefits usually involves the court e.g.
probate registry obtaining letter of Administration from probate Registry.

Claim Notification/processes in Non-Life Insurance


- Immediate Notification by the Insured to Insurer
- Police report in case of Burglary/Theft Claim or Fatal motor accident as
well as issues affecting the third party.
- No Admission of liability, except with the written or express permission of
insurer
- Right of Action: The insured must assist the Insurer in exercise of
subrogation rights against any negligent third party responsible for the
loss.
- Claims supporting documents e.g. details/particulars of loss.
- Proof of loss is also imperative.

5(a) Outline the main information contained in the schedule of a Standard Fire
and Special perils policy. (6 marks)
(b) Explain the general features of renewal procedure in Non-life Contract.
(4 marks)
Solution 5
Standard Fire and Special peril policy schedule contained the following
information
- Policy Number
- The Insured Name and Address
- The Insured occupation
- Scope of Cover
- The sum insured/value at Risk
- Period of Cover
- Annual Premium
- Description of property insured
- Clauses and warranty/extension of cover
- Declaration/signature

(b) Offer to renew (Renewal Notice should be sent to the insured by the
Insurer before the renewal date.
- Renewal Notice is not obligatory on the part of the Insurer, and the
Insured is also not compelled to renew unless Long Term Agreement
Clause is incorporated.
- If the insured accept to renew, the value of the property insured can be
reduced or increase at renewal.
- Renewal procedure is completed, once the insured pay the renewal
premium due to the Insurer, and renewal endorsement issued to confirm
cover.

6. A manufacturing company insure its Plant and Machinery valued at


N1,850,000:00 five year ago. The machinery was completely destroyed by fire,
and if the replacement cost as the time of loss is N2,500,000.00, and the life
span of the machinery is 10 years.

Briefly explain how Insurer would settle the claim

(a) On Indemnity basis (5 marks)

(b) Reinstatement value basis (5 marks)

Solution 6
(a) Sum Insured: N1,850,000:00
Value of the time of loss: N2,500,000:00

The Insurer will settle the claim on Indemnity basis based on the sum
insured and Average will apply because the sum insured is less than the
value at machinery at the time of loss. The insurer will penalize the
insured for under-insurance.

(b) Reinstatement Value basis: The Insurer will not settle the claim based on
the full value of the machinery as the sum insured less than value at the
time of reinstatement (at less than 85% of the full value) at the time of loss.

7. Explain the main factors that limit amounts to be paid at the time of
settlement of claim in General Insurance. (10 marks)

Solution 7
- Excess this is the amount which the insured bears for each and every
loss.
- Franchise Like excess once the loss is below the franchise amount, the
insurer will not pay the loss.
- Deductible this is a large excess voluntarily assumed accepted by the
insured for a premium discount.
- Average this is the penalty for underinsurance.
- Two condition of Average a form of average where the insured item is
covered by double insurance where one policy a general one while the
other is more specific.
- Adequacy/Nature of Cover if the cover is not adequate, e.g. the sum
insured is below the value at the time of the loss, the insured cannot get
indemnity.

8(a) Explain the scope of cover under Fidelity Guarantee Insurance. (6 marks)

(b) Mention the basic exclusions under Fidelity Guarantee Policy. (4 marks)

Solution 8
Fidelity Guarantee Insurance provides cover against the dishonesty or infidelity
of an insured employee which resulted in loss or damage to the insured. The
policy can be issued on a named or unnamed basis or issued on per capital basis.

The policy is issued on annual basis, and can be extended to cover loss as a result
of error or mistake.

(b) Exclusions:
- Loss or damage as a result of connivance, error or omission
- Loss of damage which are covered under a more specific policy
- Loss or damage which are specifically excluded from the policy, or caused
as a result of fortuitous event.

9 Explain the terms


(a) Adjustable Premium. (4 marks)
(b) Premium Reserve (6 marks)

Solution 9
(a) Adjustable premium is premium which is calculated based on estimates
e.g. under declaration policy where the limited premium paid is adjusted
upward or downward, after the insured submitted full declaration at the
end of the year e.g. marine or stock declaration policy

(b) Premium Reserve: In certain cases, the premium basis may not be known
at inception and therefore the insurer will offer to the proposer pending
when the actual risk details is provided.

10. Define the following Reinsurance Terms:


(i) Quota Share
(ii) Surplus Share
(iii) Excess of Loss
(iv) Excess of Loss Ratio (stop loss)
(v) Retention Limits
(10 marks)
Solution 10
Quota Share: This is a proportional treaty arrangement between the ceding
Company and Reinsurer where the primary insurer cedes a certain percentage of
the risk accepted which is above its capacity to the reinsurer e.g. 70/30%

Surplus share:
This is a proportional arrangement where the primary insurer cedes an amount
which is over and above its retention limit to the reinsurer and the reinsurer is
oblige to accept it. The retention of the insurer is called a Line and the reinsurer
takes a multiple of lines.

Excess of Loss:
This is non-proportional treaty arrangement where the ceding company pays the
first x naira of losses arising from an event, and the reinsurer pays y naira in
excess of x naira arising out of an event. This arrangement is usually on claim
basis and it is written in layers.

Excess of Loss Ration (Stop loss)


Non proportional treaty arrangement which provide reinsurer cover for an entire
portfolio of risk, rather than for individual losses i.e. when the loss ratio for a
type of business exceeds a particular figure, the reinsurer agrees to pay and the
loss ratio is claims expressed as percentage of premium.

Retain Limits
This is the amount of risk, a primary underwriter retain before ceding the
balance to the reinsurer i.e. in surplus reinsurance where a Line equal Retention
Limit.

11. Discuss the concept of Risk Management as it relates to operational risk of an


Insurance organization.
(10 marks)
Solution 11
Risk Management Concept involves the assessment of Risk, determination of the
level of risk exposures, minimize the risk of loss and also reduce the extent of
loss, and finally risk prevention and elimination i.e. Risk identification,
Evaluation, Financing and control. An Insurance organization set up a
department called Enterprise Risk Management (ERM), and the responsibility of
this department is to assess general risk exposure of the company in carrying out
its operations for instance to appraise the risk and determine the level of
exposure of the company in the overall interest of the company to declare
underwriting profit and reduce the operational risk of the organization

12. Compare the scope of cover granted by Engineering policy and Business
Interruption policy. (10 marks)

Solution 12
Engineering Insurance provide compensation to the insured in the event of
insured plant and machinery being damaged by some special perils or the plant
and machinery breakdown due to unforeseen circumstances. Engineering
Insurance required a compulsory Inspection before cover is granted. The scope of
cover granted by Engineering Insurance is summarized as follows:
- Damage to or breakdown of specific items of plant and machinery
- Cost of repairs or damage to surrounding property
- Legal liability for injury caused as a result of breakdown of plant and
machinery
- Legal Liability for damage caused to adjourning property,

However loss of profit Insurance covers loss of profit resulting from damage to
physical property by fire and special peril risk, and this loss not directly related
to physical damage but connected to additional costs incurred during the repairs,
period and maintenance of plant, and machinery are covered under
consequential loss.
- The basic headings are:
(a) Cost of overhead expenses
(b) Loss of Profits as a result of damage to physical property
(c) Increased in cost of running the business (Temporary accommodation
while the property is being repaired.

13. Discuss the function of Loss Adjuster and Risk Control Surveyor in handling
Non-life Insurance Claim. (10 marks)

Solution 13
Loss Adjuster
- Loss Adjusters are independent claim investigation experts appointed by
the insurers to investigate, adjust and make final recommendation to
insurers in order to determine the insurer liability as regard general
insurance claims.

- The loss Adjuster will check if the policy is in force at the time of loss, and
whether cover is adequate or not. Also, they will check if the peril that
caused the loss is covered under the policy, and if all the policy clauses
and warranty are fully complied with and adhere to. They also ensure that
premium was paid before the loss.

- The loss adjuster after investigating a loss prepares a preliminary report in


case of large claim and subsequently forwards a final report to the
underwriter for consideration.

Risk Control Surveyor


The surveyor will prepare a report for the underwriter and in the case of many
property risks, will also draw a plan. The report will cover a number of features,
including the following:
A full description of the risk.
An assessment of the Level of risk
A measure of the maximum probable loss (MPL)

14. Define the following terms:


(i) Average
(ii) Excess
(iii) Franchise
(iv) Deductible
(v) Two condition of Average
(10 marks)

Solution 14
Average:
This is the penalty for under insurance. The insured is made to contribute for the
cause of the loss.

Excess:
Where the insured bear the first part of a loss e.g. if the excess is N1,000 and the
loss is N10,000.00, the insurer pay N9,000 and insured bears N1,000.

Franchise:
Is the amount which the insured bears for a loss provided it is not more than the
franchise amount. Where the loss is more than the franchise amount the insurer
will pay for all the loss.

Deductible:
This is a large excess voluntarily assumed by the insured for a reduction in the
premium he will pay for an insurance cover.

Two Condition of Average:


A form of average where the insured item is covered by double insurance where
one policy a general one while the other is more specific.

15. Briefly explain what you understand by the term Co-reinsurance.


(10 marks)
Solution 15
This operates where two or more insurance companies as a result of the complex
nature of the risks, e.g. Oil and Gas risk, Aviation risk, decides to do risk
scheduling i.e. share the risk in agreed ratable proportion depending on the size
of the risk, many insurance could be involved. Premium due on it is equally
ratably apportioned.

16. Enumerate the importance of Insurable interest in the contract of Insurance.


(10 marks)
Solution 16
Insurable interest is legally recognized relationship between the insured and the
loss e.g. the ownership of the property, and the importance of insurable interest
is very critical to Insurance contract, Hence any insurance contract without
insurable interest is unenforceable at law.

The main importance is that it prevents those without insurable interest in a risk
from buying insurance cover thereby preventing fraud that could have arisen
from such form of transaction.

17. Outline the major features of Life Assurance Contract. (10 marks)

Solution 17
- Life Assurance contract is permanent policy
- The occurrence of a claim would definitely occur either through death or
maturity.
- Premium calculation is derived from mortality table and premium paid is
mostly level throughout the duration
- Life Assurance policy can be cash-surrendered i.e. it has surrender value
- Insurable interest must exist in life assurance contract
- Participation in profits: After valuation of Assets and liabilities, any
surplus existing would be distributed to policyholder inform of profits,
and this is usually applicable to with-profits endowment policy.
- Paid-up policies: Alternative to surrender value a lower sum assured is
paid at maturity.

18. Compare the scope of cover granted under Group Life Assurance and Group
Personal Accident Cover. (10 marks)

Solution 18
- Group Life Assurance policy is usually effected by the employer for the
benefit of his employer who may die while in service or during the course
of employment, while the Group Personal Accident policy could be
effected by employer, or society to cover members against accidental
deaths or bodily injury occurring anywhere in Nigeria, whether at work or
outside work.

- Group Life covers death resulting from Natural cause, while Group
Personal Accident covers death resulting from accidental cause.

- Group Life is underwritten by life office, while Group Personal Accident is


underwritten by Non-Life office.

19(a) Differentiate between Self-Insurance and Captive Insurance. (6 marks)

(b) Explain how Subrogation is a Corollary of Indemnity. (4 marks)

Solution 19
(a) Self Insurance is a situation where an organization set aside funds to
mitigate or settle any losses that may arise from its operation while captive
Insurance is when an organization set-up a subsidiary or affiliate company
to insurer the assets of the parent company.

(b) Subrogation is a corollary of indemnity, because under common law, when


an Insurer indemnifies an Insured, it is permitted to take over the rights of
the Insured to recover any amount payable from other parties who might
be liable for the loss. The essence is to ensure that the Insured claim from
both the insurer and tortfessor and thereby making a profit from his loss.

20. Outline the major regulatory functions performed by NAICOM. (10 marks)

Solution 20
Regulatory function performed by NAICOM
- They process and approved application for registration by new Insurance
Company and issue operating license to new company.
- They ensure that Insurance companies meet the solvency margin
requirement as stipulated in NAICOM Act, 2003
- They carried out periodic inspection of insurance company operation and
activity.
- They ensure that insurance company comply with the periodic valuation
of Assets requirement to determine any surplus in their funds or
insolvency
- They ensure that all Insurance company remits the 1% Insurance
supervisory levy to the Commission as stipulated in the Act.
- They ensure that companies meet the requirement for sending Annual
Returns for approval in line with IFRS etc.
- Protect the policyholder interest in the area of claims disputes, and delay
in claim settlement
- Grant renewal approval the operation licence of Insurance companies
D04

LOGO

CHARTERED INSURANCE INSTITUTE OF NIGERIA


DIPLOMA

OCTOBER 2013 EXAMINATION PAPER

SUBJECT D04
BUSINESS PRACTICE

Three hours are allowed for this paper

Fill in the information requested on the answer booklet and on form B.

Do not write on the question paper.

The answer booklet and this question paper must be handed in personally
by you to the invigilator before you leave the Examination Hall. Failure to
do this may result in your paper not being marked and you may be
prevented from entering for this examination in future.

D04
CHARTERED INSURANCE INSTITUTE OF NIGERIA

D04 BUSINESS PRACTICE

INSTRUCTIONS TO CANDIDATES

Three hours are allowed for this paper which contains 20 compulsory questions
and carries a total of 200 marks.

Where a question is split into parts (a), (b) etc, marks for each part are only
shown if they are split unevenly between the parts and you should spend your
time in accordance with the allocation.

Answer each question on a new page. If a question has more than one part,
leave several lines blank after each part.

It is important to show each step in any calculation, even if you have used a
calculator.

You may find it helpful in some places to make rough notes in the answer
booklet, if you do this, you should cross through these notes before you hand in
the booklet.
D04 BUSINESS PRACTICE

1(a)i. What do you understand by the term bartering? (3 marks)

(a)ii. What is the major problem associated with bartering? How can it be
overcome?
(4 marks)

(b) List the three (3) most important areas in the development of business
regulation.
(3 marks)
Solution 1
a(i) Bartering is the exchange of one set of goods or services for another. Each
type of goods or services has a value that can be measured in terms of the
other. (3 marks)

(ii) The main problem of bartering is that it relies on the so-called double
coincidence of barter; each party in a barter transaction must want what
the other party has.
(3 marks)

This problem would be overcome by the use money.


(1 mark)

b. The three most important areas in the development of business regulation


are:

1. Laws governing contracts;


2. Laws governing property rights;
3. Incorporation of companies
1 mark each = 1 x 3 (3 marks)

2(a) Explain what you understand by an organizations vision statement. Give one
(1) example.
(4 marks)

(b) List six (6) principal functions of a business.


(6 marks)

Solution 2
(a) A vision statement is a description of where an organization wants to be at
a stated time in the future. It may deal with issues of future ownership,
broad financial performance goals, future markets where the organization
wants to operate.

Explanation 3 examples = (4 marks)

(b) Principal functions of a business are:

1. Innovation or Research and Development


2. Production
3. Marketing
4. Human Resources Management
5. Finance and Accounting
6. Compliance with regulations

1 mark each for 6 points = 1 x 6 = (6 marks)

3. Outline the main characteristics of matrix culture. (10 marks)

Solution 3
The main characteristics of matrix culture are:

(1) Resource allocation and goal-setting decisions are made bureaucratically


(2) Operational decisions are made on a project group basis.
(3) Specialist staff are used across the whole organization
(4) The organization relies on a team system
(5) There is a cross-disciplinary approach
(6) A more stimulating work environment is provided for individual workers

4. List the three (3) levels of information and discuss each of them. (10 marks)

Solution 4
The three levels of information are:

1. Strategic Information: is used by senior managers to plan the objectives of


their organization and to assess whether the objectives are being met in
practice. Such information includes: overall profitability, the profitability
of different segments of the business, future market prospects, the
availability and cost of raising new funds, total cash needs, total meaning
levels and capital needs. Strategic information will be used for the
management decision making described as strategic planning.
(4 marks)

2. Tactical Information: This is used by middle management to ensure that


resources of the business are employed to achieve the strategic objectives
of the organization. Such information includes productivity control or
variance analysis reports and cash flow forecasts, manning levels and
profit results within a particular department of the organization. A large
proportion of this information will be generated from within the
organization and is likely to have an accounting emphasis.

It is usually prepared regularly and it is used for the decision making


referred to as management control.
(4 marks)

3. Operational Information: is used by front-line managers such as


supervisors to ensure that specific tasks are planned and carried out
properly and effectively. (2 marks)

5(a) List the three (3) statutory functions of the Corporate Affairs Commission
(CAC). (3 marks)
(b) State three (3) statutory registers to be maintained by a Companys Secretary.
(3 marks)
(c) List the four (4) components of an Annual Account. (4 marks)

Solution 5
(a) The three statutory functions of the CAC are:
i. To incorporate and dissolve companies
ii. to examine and hold documents under the companies net and related
legislation and
iii. to make this (i) and (ii) above information available to the public
1 mark each = 1 x 3 = (3 marks)

(b) Statutory registers are:


i. register of members
ii. register of directors and secretaries
iii. register of directors interest
iv. register of charges
v. register of interests in shares (for public companies)
1 mark each for any 3 points = 1 x 3 = (3 marks)

(c) Components of annual accounts include:


i. a profit and loss account
ii. a balance sheet signed by a director
iii. an auditors report signed by the auditor
iv. a directors report signed by a director or the company secretary

1 marks each = 1 x 4 = (4 marks)

6. Distinguish between:
(a) budgeting and forecasting; (3 marks)
(b) bottom-up and top-down budgeting; and (3 marks)
(c) financial accounting and management accounting (4 marks)

Solution 6
(i) Budgeting is usually concerned with relatively short periods of time, up to
year, whereas forecasting looks forward to around three years or longer.
(3 marks)

(ii) Bottom-up budgeting indicates that more initiatives in the process comes
from individuals and departments in the formulation of an agreed budget
whereas Top-down budgeting denotes that the main outlines and
decisions are made by the executive.
(3 marks)

(iii) Financial accounting is essentially backward looking and the financial


accounts produced are presented for external use i.e. to inform the
stakeholders how well the company has been run over the last accounting
period.
(2 marks)

Whereas management accounting looks at present financial performance


and projects forward to the future to enable managers make sound
decisions relating to the business. It is for internal use only.

7. Why are the following interested in a companys financial information?


(a) Directors and Managers (2 marks)
(b) Public (2 marks)
(c) Competitors (2 marks)
(d) Brokers (2 marks)
(e) Customers (2 marks)
Give two (2) reasons for each of them.

Solution 7
(i) Directors and managers: have overall responsibility for managing the
business. Its members may need to know whether or not the organization
has been making the best use of its resources. In addition, they may want
to know whether there has enough money to expend its operations,
whether some parts of the business are more successful than others, and
whether the company has behaved as a responsible part of the community.
(2 marks)

(ii) Public: the public includes people who may be potential investors or
shareholders in the organization, pressure groups that may want to
monitor aspects of the organizations activities, and people who might be
considering applying to work for the organization.
(2 marks)

(iii) Competitors: An organizations competitors can use financial information


about it to help them understand its strengths and weaknesses readily.
(2 marks)

(iv) Brokers: From an insurance point of view, business will want to know
whether companies they deal with are financially strong.
(2 marks)

(v) Customers: potential and existing customers of insurance companies want


to know that they are insured with a reputable organization that is able to
pay its claims in the event of a loss.

8. Outline the new skills that Managers must possess to fit into the new
changing working patterns.

Solution 8
The new skills include:
1. Earning the respect of subordinates and colleagues: in line with a gradual
move towards democratization, a manager is more likely to be answerable
to their subordinate now than the past. This means that there a much
emphasis today on earning the respect of colleagues by developing
interpersonal skills and empathy with other staff.
(2 marks)

2. Focusing on customers: The way in which organizations now have to


regard the customer means that managers must be aware of the impact of
their decisions have on existing and potential client-bases. Successful
business managers acknowledge that their management policies must now
be customer focused, and that they have to act on feedback from their
clients and customer is essential.
(2 marks)

3. Multi-tasking: Managers need to be able to carry out administrative


functions quickly and easily themselves. Added to this, managers today
need to know the fundamental concepts behind information technology
(IT), finance, accounting, human resources and sales and marketing, in
addition to having a deeper knowledge of their own specialized areas.
(2 marks)

4. Mobility and global outlook: many organizations trade internationally or


with regional trading partners and managers are expected to be able to
travel and conduct business in different economic, cultural and financial
environments. Also in an increasingly deregulated commercial world, it is
important for managers to be able to identify the advantages of operating
in other countries, and ways of entering new markets.
(2 marks)

9(a) Explain the concept of Human Resources as a business component.


(5 marks)
(b) Explain the five Cs of decision making. (5 marks)

Solution 9
(a) Human Resources are made up of the people who work for the
organization on a permanent, temporary, full-time or part-time basis and
those who work directly or indirectly for the company in an outsourced
capacity. The management of people has become an increasingly
important management skill.

(b) Five Cs of decision making are;


1. Consider: the preparation stage at which the problem is considered
2. Consultant: the stage at which initiatives are taken to involve those
affected

3. Crunch: the need to ensure that something is done

4. Communicate: the stage at which what has been decided and why, is
explained to staff.

5. Check: the need to go back and monitor the results of the decision
1 mark each = 1 x 5 = (5 marks)

10(a) Itemize six (6) main components of managing an Organization that are
affected by its structure. (6 marks)
(b) List two (2) main activities of WTO. (4 marks)

Solution 10
(a) The six main components affected by our organizations structure are:
1. Ease of internal communication
2. Individual freedom to make decisions
3. Promotion of innovations
4. Degree of staff specialization
5. Ability to grow
6. Flexibility in the face of change
1 mark each = 1 x 6 = (6 marks)

(b) Four main activities of WTO are:


1. Establishment of rules which govern trading behavior between
nation states;
2. Attempts to liberalize trade through tariff and quota reduction;
3. Offering a legal framework and a court for settlement of disputes
4. Monitoring trade agreements and government policies, especially
those that may contravene the WTO objective of the liberalization.
1 mark each = 1 x 4 (4 marks)

11. With the use of a diagram distinguish between a vertical and flat
management structure.
(10 marks)
Solution 11
A vertical management structure
Chief Executive Officer (CEO)
Deputy CEO

Senior Manager

Manager

Junior Manager

(2 marks)
Assistant

In vertical management structure there is a clear demarcation of the authority to


take decisions, with authority increasing as position claims higher up the ladder.
In vertical structured organization, even routine decisions may need to be
approved by more senior manager. (3 marks)
A flat management structure
CEO

Operators Finance Personal Marketing


Director Director Director Director

(2
marks)

In a flat structure, decision making authority is delegated evenly across a broad


range of managers and there is less need to refer routine management decisions
to move senior managers as shown overleaf. (3 marks)

12. Outline ten (10) statutory rights that cannot be signed away by the employer
or employee in a contract of employment. (10 marks)

Question 12
The statutory rights that cannot be signed away by the employer or employee
are:

1. equal pay with members of the opposite sex providing it is likework or


equal value.
2. maternity and paternity aspects and benefits
3. Parental leave
4. an itemized pay statement
5. Not to have any unlawful deductions from an employees pay
6. be paid when an employee is laid off
7. redundancy pay (subject to service conditions)
8. part time/contract workers to be treated in the same manner as full time
employees
9. statutory sick pay
10. paid time off for certain duties
11. belong or not belong to a trade union
12. a safe system of work
13. not to be unfairly dismissed
14. not to be discriminated against on grounds of race, sex, marriage or
disability
15. employee access to their personal data held by the employer.
1 mark each for any 15points = (10 points)

13(a) Explain the term accounting equation.

(b) Define the following terms:


(i) Solvency margin (2 marks)
(ii) Tangible Assets (2 marks)
(iii) Depreciation (2 marks)
Solution 13
(a) Accounting equation: This is the relationship between the things owned by
a business (its) assets) and the funds which were used to buy them is
expressed by the accounting equation:

Assets = Capital Liabilities or


Capital = Assets + Liabilities
(4 marks)
(b)i Solvency margin is a strange indication of an insurance companys
strength and reserves, the protection afforded to conditions, and
ultimately, its ability to pay claims to its customer. (2 marks)

ii Depreciation: is the writing-down of the value of an asset over a period of


time in the organizations balance sheet, to reflect the cost of its use.
(2 marks)

iii. Tangible Asset: is one that is physical i.e. can be touched such as cash,
land, buildings, machinery or investments. Some tangible assets especially
machinery and equipment lose their value as time goes by.
(2 marks)

14(a) List five (5) tasks that are commonly addressed by Public Relation. (5 marks)
(b) Itemize five (5) areas that a market planning should incorporate. (5 marks)

Solution 14
(a) The tasks mostly common addressed by PR are:

1. motivating the organisations workforce by fostering pride in its public


face.
2. handling a specific problem or issue
3. building or maintaining the corporate image
4. influencing particular audiences or market segment
5. supporting other communication activities e.g. an advertising campaign
6. assisting in the launch of a new product or service
1 mark for any 5 points = (5 marks)

(b) Market planning will incorporate

1. allocation of responsibilities
2. sales targets by area, outlet
3. budgets
4. timescales
5. delivery and availability of the goods or services according to the launch
date
6. feedback on progress of sales in relation to the target.
7. arrangements for receiving and responding to customers complaints, and
returns or replacement policy.
1 mark for any 5 points = 1 x 5 = 5 marks
15(a) Define Marketing. (4 marks)
(b) Explain what is meant by a Market Segment. (6 marks)

Solution 15
(a) Marketing is a series of management processed based on a person or
organisations interaction with, and understanding of, existing and
potential customers, so as to provide them with what they need and want.
(4 marks)

(b) A market segment is a group of people with relatively uniform demands


and needs compared with the rest of the market. Each segment contains
people who are similar in their needs and wants, and in the product
benefits they seek. Each segment seeks a different set of benefits from the
same product category, and some segments may be broken down further
into sub-segments. (6 marks)

16. Explain with example the five (5) methods of training. (10 marks)

Solution 16
Methods of training are:

1. On the job: is the training actually conducted at the workplace e.g.


showing someone how to do a task and then supervising them until they
are proficient. (2 marks)

2. Off the job: is training conducted away from the workplace e.g. classroom
training, internal company houses or workshops and external courses that
are carried out away from workplace. (2 marks)

3. Open learning: is a structured self-training approach utilizing training


materials and books. It has the benefit that the trainee can work on the
training whenever they want to, either inside the workplace or outside - at
home. (2 marks)

4. Computer Based Training (CBT) involves the use of PC computers with


CD-ROMS with prepared training programmes i.e. e-learning.
(2 marks)
5. Professional certificates or Diplomas e.g. CIIN examinations.

These examinations are a learning experience and cover a number of topics


that are important to developing competence in the profession concerned.
( 2 marks)

17. Explain the following cultures as identified by Handy:


(a) The Power Culture (2 marks)
(b) The Role Culture (2 marks)
(c) The Task Culture (2 marks)
(d) The People Culture (2 marks)

Solution 17
1. The Power culture-information is kept at the centre and people outside the
centre are played off against each other. Such a culture tends to win/lose
fights and power struggles, but it is often held together by a charismatic
leader in the centre.
(2 marks)

2. The Role Culture: This is essentially a bureaucratic model in which


organizational values and procedures are highly certified. Everyone knows
their place and there are tough sanctions should they step out of line.
(2 marks)

3. The task culture: In this culture there is extensive use of transient project
teams. People are respected for their energy and contribution rather than
their status.
(2 marks)

4. The people culture: The individual is the focus in this organization which
can appear anarchic. This culture is common in start-up situations but
can lead to excessive conflict in an established business.
(2 marks)

18. Identify eight (8) signs of a poorly motivated workforce. (10 marks)

Solution 18
Signs of a poorly motivated workforce include:

1. abnormally high turnover of personal;


2. loss productivity;
3. absenteeism
4. Poor quality of work;
5. persistent lateness;
6. couldnt care less attitude to work;
7. fixed ideas and excessive resistance to change;
8. lack of acceptance of responsibility;
9. a plethora if petty complaints and grievances with little or no foundation

1 for any 8 points = 1 x 8 = (10 marks)

19(a) Explain the stakeholders theory. (5 marks)

(b) List five (5) actions that are needed by management to appreciate the
stakeholders. (5 marks)

Solution 19
(a) Stakeholders theory relates to the fact that as stakeholders exist and can
apply considerable influences on a company, management will have a key
task of having to balance the requirements of the various stakeholder
groups. They will need to develop good relations with the groups, create
audiences, and use persuasion and influencing skills to ensure
stakeholders needs are as near as possible met.
(5 marks)

(b) Actions by arrangement to appreciate stakeholders include:

1. identify the businesss stakeholders, who they are:


2. find out their expectations, the minimum return each stakeholder will
accept;
3. identify the strength of each stakeholders influence on the way the
company acts;
4. establish who the key people are in each stakeholder group and develop
good relations with them
5. identify the views and attitudes of the various stakeholders to the
companys mission, strategies, activities and where necessary involve any
change;
6. establish the stakeholders who support the managements business
policies and approval, and find out those who do not agree;
7. find out how antagonistic stakeholders can be won over, use supporters to
exert their influence on antagonists.

20(a) Differentiate between ratio forecasting and synthetic forecasting. (4 marks)

(b) Itemise six (6) problems that are associated with manpower planning.
(6 marks)
Solution 20
(a) Ratio forecasting needs are calculated from a fixed relationship between
manpower requirements and other factors (e.g. sales volumes, policies
issued) while in synthetic forecasting needs are built up from forecasted
output in terms of estimated work load (e.g. output expressed in terms of
man hours).
2 marks each = 2 x 2 (4 marks)

(b) Problems and limitations with manpower planning are:

1. obtaining accurate forecast


2. obtaining the information (and updating)
3. rapid changes in the business environment
4. legislation
5. changes in the labour markets
6. changes in the economy
7. government policies
8. difficulty in sustaining commitment
1 mark each for any 6 points = 1 x 6 = (6 marks)
D05

LOGO

CHARTERED INSURANCE INSTITUTE OF NIGERIA

DIPLOMA

OCTOBER 2013 EXAMINATION PAPER

SUBJECT D05
INSURANCE LAW

Three hours are allowed for this paper

Fill in the information requested on the answer booklet and on form B.

Do not write on the question paper.

The answer booklet and this question paper must be handed in personally
by you to the invigilator before you leave the Examination Hall. Failure to
do this may result in your paper not being marked and you may be
prevented from entering for this examination in future.
D05

CHARTERED INSURANCE INSTITUTE OF NIGERIA

D05 INSURANCE LAW

INSTRUCTIONS TO CANDIDATES

Three hours are allowed for this paper.

You should answer all questions in Part 1 and two (2) out of the four (4)
questions in Part II.

The paper carries a total of 200 marks distributed as follows:


Part I - 140 marks.
Part II - 60 marks.

You are advised to spend not more than two (2) hours on Part I. You are
strongly advised to attempt ALL the required questions to score maximum
possible marks.

In attempting the questions, you may find it helpful in some places to make
rough notes in the answer booklet. If you do this, you must cross through these
notes before you hand in the booklet.

Answer each question on a new page. If a question has more than one part
leave several lines blank after each part.

D05
PART I
Answer ALL questions in Part I.
Each question carries 10 marks.
Note form is acceptable where this conveys all the necessary information.

1(a) Define Insurable Interest. (3 marks)


(b) Why does the law require insurable interest. (2 marks)
(c) Give at least 5 (five) examples of people who may have insurable interest
in property. (5 marks)

Solution 1

(a) Insurable interest is the legal right to insure arising out of a financial
relationship recognized at law between the insured and the subject matter
of insurance.

It means the policy holder must be in a position where he will suffer if the
risk he insured against were to occur.

(b)
i) To reduce moral hazard
ii) To discourage wagering

(c)
i. Outright owners of property
ii. Part or joint owners
iii. Mortgagees and mortgagors
iv. Executors and trustees
v. Landlord and tenant
vi. Bailees
vii. People living together
viii. Finders and people in possession

2. It may be argued that no formality or any form of written documentation


is required in the formation of an Insurance contract. In fact, an insurance
cover may be given orally. Give at least five (5) exceptional cases to this
statement.
(10 marks)

Solution 2
i. Contract by deed
ii. Insurance contracts which must be in writing
iii. Insurance contracts which must be evidence in writing
iv. Other insurance contracts where written documentation is required e.g.
motor insurance
v. Life insurance contracts.

3(a) In relation to material fact that needs to be disclosed, define briefly:


(i) Physical Hazard (2 marks)
(ii) Moral Hazard (2 marks)
(b) Give at least three (3) examples of each. (6 marks)

Solution 3
(a)
i. Physical hazard
This relates to facts bringing out adequate description of the subject matter
of insurance usually contained in the proposal form and if no Form is
used, in survey of the risk.

ii. Moral hazard


This relates to any feature of the subject matter of insurance e.g. non-
standard construction in the case of buildings which makes the risk worse
than a normal risk of its class.

(b)
i. Physical Hazard
Fire insurance construction of the building and fire fighting
equipment.
Theft insurance the nature of stock, its value and nature of security
precautions
Life insurance age, previous medical history.

ii. Moral hazard


Identity of the insured
Criminal acts
Any other adverse insurance history
Details of other policies currently in force

4.(a) Briefly explain the doctrine of Privity of Contract. (2 marks)


(b) List the well established exceptions to the principle in the field of
Insurance. (8 marks)
Solution 4
(a) Privity of contract is a doctrine which restricts the rights and duties created
by a contract to the persons who originally made it. For example, a
contract between A and B cannot confer any legally enforceable benefit on
a third party and cannot impose duties on the third party.
Only a party to a contract can sue upon it;

(b)
i. Agency
ii. Trusts
iii. The Road Traffic Act 1988, Section 148 (7) and 151
iv. The Third Parties (Rights against Insurers) Act 1930
v. Law of Property Act 1923, Section 47
vi. Fire Prevention (Metropolis) Act 1774, Section 83
vii. Policies with Additional Insureds
viii. Noting the interest of third parties.

5. With the aid of decided cases, briefly explain how Mistake can void an
Insurance Contract. (10 marks)

Solution 5
In the ordinary way, mistake occurs where the contracting parties are working at
cross purposes or labouring under mistaken belief about a particular state of
affairs. The effect is to void the contract of the parties.

Mistake will void insurance contract if it is fundamental and goes to the root of
the contract. The following cases illustrated this principle.
Strickland v Turner a contract to purchase an annuity was void for mistake
because the annuitant had died before the date of purchase.
Scott v Coulson a policy on the life of a person names Mr. A. Death was sold
for 460 on the assumption that he was alive. In fact, Death was dead and the
maturity value of 777 was far greater. The vendor succeeded in having the
contract to sell the policy set aside.
In Beach v Pearl Assurance, the parties worked at cross purposes. In the case the
Proposer wished to insure the life of her mother, Mary Ellen Ince. The insurer
agent thought it was meant for her grandmother, Mary Ellen Ince. The policy
was issued in the name of Mary Ellen Ince but the details were appropriate and
premium calculated on the basis of the grandmothers age. Claim made upon the
death of the mother was dismissed because no consensus ad idem between the
parties.
6(a) Give a simple definition of Subrogation. (2 marks)
(b) Why does the law allow subrogation? (2 marks)
(c) Will an Insurer who made an ex-gratia payment be entitled to
Subrogation? (1 mark)
(d) In what three (3) ways can Subrogation arise? (3 marks)

Solution 6
(a) The right of one person having indemnified another under a legal
obligation to do so, to stand in the place of that other and avail himself of
all the rights and remedies of that other, whether already enforced or not.
(b) It is to prevent the insured from making a profit from their loss and
achieve more than an indemnity. In case of double insurance, right to
claim twice is denied. Loss is shared under the principle of contribution.
(c) Where the insurer makes an ex-gratia payment, it is made merely as a
favour, the insurer will not be able to subrogate against a third party.

(d)
In tort
In contract
Under statue

7. Under what circumstances may the operation of the doctrine of indemnity be


extended? (10 marks)

Solution 7
Cover or a reinstatement
New for old cover
Agreed value cover
Partial losses under undervalued policies

8. What is the relevance of Insurance to the tort of:


(a) Trespass (5 marks)
(b) Nuisance (5 marks)

Solution 8
(a) Trespass:
Trespass is an unjustifiable interference by a person with the person, land
and goods of another. The interference must be direct, intentionally and
actionable per se.

Trespass is also referred to as an intentional tort. No liability flows from it


unless there is an intention.

Insurance, unlike the tort of trespass deals with fortuitous losses-accidental


losses. An action that is self-willed will not be remedied by insurance.
However, there are some insurance policies that cover risk of trespass to
person and goods e.g. public liability policies issued to contractors which
cover risk to property of third parties. Nothwithstanding, such policies do
not cover damage that is deliberately caused.

(b) Nuisance
An unjustifiable interference with a persons right of enjoyment of his
land, or an act that is likely or actually causes inconvenience or annoyance
to the public or a section of the public and an interference with a right
common to all.

Nuisance is not of much relevance to insurance. Majority of liability


insurance claims are founded on negligence and not nuisance.

However, work done by builders and other contractors can sometimes


cause damage to adjoining property that leads to claims in nuisance. See
GOLD v PATMAN & FORTHERINGHAM. This case led to the
development of a new insurance policy called 21 2.1 Cover to protect
people who employ contractors from claims by owners that is nearby
purportedly damaged as a result of work.

Save for this, nuisance is not that relevant to insurance. Insurance policy
will respond to concrete claim or injury, noxious smell, noise which are not
concrete damage.

9(a) In relation to the Law of Contract, define an offer. (2 marks)

(b) An offer does not remain open indefinitely. List at least, four (4)
circumstances in which an offer may end. (8 marks)

Solution 9
(a) An offer in law of contract is a definite expression of intention by a party
called the offeror to do something for another called the offeree and to be
bound by it.

(b)
i. A time limit or a reasonable time
ii. Death of either offeror or offeree
iii. Acceptance
iv. Revocation of the offer
v. Rejection or counter offer

10a. Explain each of the following terms and phrases:


(i) Deductible (2 marks)
(ii) Franchise (2 marks)
(iii) A Valued Policy (2 marks)
(iv) Betterment (2 marks)

b. Distinguish between an Excess and a Franchise. (2 marks)

Solution 10
(i) Excess or Deductible
This is usually inserted in an insurance policy as a clause and it usually
provides that the insured must bear the first amount of any loss
expressed either as a sum of money or a percentage of the loss.

It may be aggregate excess which applies not to individual losses in any


one period of insurance.

The effect of an excess is to relieve insurers of having to deal with small


losses (where handling costs are likely to be high in relation to the
amount claimed) and to reduce the size of every claim payment which
is made. On the long run, the overall cost of insurance which benefits
both insurer and insured.

Excess may either be compulsory or requested by the insured in order


to gain a reduction in premium.

(ii) Franchise
Similar to excess in that there is no liability for any loss which is less
than the franchise figure. Once franchise is exceeded, the loss becomes
payable in full.
It is not as common as excess.

(iii) Valued Policy:


This type of Policy modifies the operation of the principle of indemnity.
Parties to this policy agree that in the event of a loss, a particular sum,
fixed at the onset of the insurance will be paid, regardless of the actual
value of the property at the time.

Usually, claim under a policy of indemnity is unliquidated whereas a


claim under a valued policy is liquidated.

(iv) Betterment:
This relates to property insurance. It is an improvement that increases
the value of real property especially an enhancement that goes beyond
repair or restitution to a former condition. Betterment may result from
covers for reinstatement and New for Old.

(b) Distinguish between Excess and Franchise


Franchise is not so common because it tends to introduce an element of
moral hazard. For example, if an insured a loss of N9, 500.00 and the
policy is subject to a N10,000 franchise, they might be tempted to inflate
the loss to say N10,500 in order to be paid in full. Whereas if the policy was
subject to a N10,000 excess, there would be little or no point in taking the
risk of doing so as he will recover no more than N500.

11(a) Define Equity. (2 marks)


(b) Give three (3) examples of legal principles of remedies derived from
Equity. (6 marks)
(c) Briefly explain how Equity serves as a source of law. (2 marks)

Solution 11
(a) Equity is a collection of rules which were developed to remedy some of the
shortcomings of the common law.

(b)
Law of Trusts
Specific Performances
Injunctions

(c) Equity serves as a source of law in the sense that it is regarded as the other
side of the common law. The two exist side by side. Our courts are both
courts of Law and Equity. Both common Law and Equity were both
received together as part of English law which Nigeria has been operating
for over a century.

12(a) Who is a Minor in Law? (1 mark)


(b) Explain the nature of his rights with regard to:
(i) Law of tort (3 marks)
(ii) Property law (3 marks)
(iii) Criminal law (4 marks)

Solution 12
(a) A minor in law or an infant is a person under the age of 18 years.

(b)
i. Law of tort
He is fully responsible for his actions. He can be liable in tort as an adult. He
cannot be sued directly but through his Guardian ad litem.

ii. Property law


He can own personal property e.g. clothing, books, sport equipment or a car,
but may not own legal estate in land. He can own a house outright but his
interest is beneficial only.

iii. Criminal Law


For the purpose of this, minor is categorized:
Over age 14 - Full criminal responsibility attaches to a minor
Ages 10 14 There is a rebuttable presumption that minor is incapable of
criminal intent.
Ages 1 10 There is a rebuttable presumption that minor is incapable of
committing crime.

13(a) What advantages does incorporation confer on a company registered


under the Companies and Allied Matters Act?
(6 marks)
(b) List the types of companies that may be formed under the Companies And
Allied Matters Act. (4 marks)

Solution 13
(a)
Separate legal personality
Limited liability
Perpetual existence
Access to greater capital
Company must not act ultra vires.

(b)
Public limited companies
Private limited companies, and these may be:
o Companies limited by shares
o Companies limited by guarantee
o Unlimited companies

14(a) Define Negligence. (2 marks)

(b) What are its major components. (3 marks)

(c) List at least five (5) defences in law against the tort. (5 marks)

Solution 14
(a) The omission to do something which a reasonable man, guided upon those
considerations which ordinarily regulate the conduct of human affairs,
would do or doing something a prudent and reasonable man would not
do.

(b)
Duty of car owed by the defendant to the claimant
A breach of that duty by the defendant
Damage suffered by the claimant as a result of the breach.

(c)
Self defence
Necessity
Act of God
Volenti non fit injuria
Contributory negligence

Part II
Answer Two (2) out of the following Four (4) questions.
Each question carries 30 marks.

15. Discuss, with relevant examples and decided cases where applicable, the
application of the doctrine of Insurable Interest with respect to the
following:

(a) Business Relationships (10 marks)


(b) Insurance of Profits (5 marks)
(c) Liability Insurance (5
marks)
(d) People with limited interest in Property (10 marks)

Solution 15
(a) Business Relationships:
i. Partnership each partner has insurable interest in the life of the other.
ii. Employer and employee Employer can insure the life of his employee to
the extent of the employees salary. This principle applies to personal
contract.
iii. Creditor and debtor a creditor has insurable interest in the life of his
debtor. But a debtor has no corresponding interest in the life of his
creditor. See HEBDON v WEST.

(b) Insurance of Profits:


An example is business interruption insurance and it covers profit which
is lost following damage to the insureds property e.g. factory buildings,
machinery and other goods. This is not a mere expectancy but a policy
founded on a legal right of ownership of the property and this gives
insurable interest. See BARCLAY V COUSINS.

(c) Examples are part owner of property, mortgagees. Their interest is limited
to the extent of the value of their share in the property and the amount of
the loan, respectively.

A person with limited interest in property may insure the property for its
full value. But he cannot keep the whole of the insurance money. The
maximum he can get from it is the value corresponding to his interest. He
will hold the balance on trust for the other person(s) who has interest. So
also is a Bailee. See WATERS V MONARCH FIRE AND LIFE
INSURANCE COY.

16(a) Identify the two (2) categories of principles of interpretation which the
courts use to interpret the words in policy documents when resolving
disputes arising out of Insurance contracts.
(b) Outline the common Law rules of interpretation of the words in Insurance
policy documents. (10 marks)
(c) Explain with the aid of decided cases, the following:
(i) How ambiguities in an Insurance policy may be resolved by the court.
(10 marks)
(ii) How inconsistencies and contradictions in the policy document may be
resolved by the court.
(6 marks)
Solution 16
(a)
i. Statutory Rules
ii. Common Law Rules

(b)
i. Ordinary Meaning
ii. Technical or Legal Meaning
iii. Contextual Meaning
iv. The Contra Profertem Rule

(c)
i. Words used in a policy may have two or more possible meanings which
either the insured or the insurer may differently hold on to. When this
happens, the court may adopt the contra profertem rule to resolve the
logjam.

The rule states that any ambiguity in any word of a Policy will be
construed against the party that inserts it or who wants to take advantage
of it. In this case, it is the insurer being the drafter of the policy. It will be
the reinsured in a reinsurance contract. See HOUGHTON V TRAFALGAL
INS. COY.
For a successful operation of the rule, there must be genuine ambiguity i.e.
there must be alternative construction that is reasonable rather that one
which is grammatically possible but farfetched in reality.

ii. Inconsistencies and contradictions may also occur in an insurance policy


so that one part of the document appear to conflict with another. The
courts have developed the ways out as follows:

where printed words conflict with words that are handwritten or typed,
the latter takes precedence since it is assumed that the parties intended to
adopt a standard form to meet the needs of their particular case. An
endorsement is likely to overrule anything in the printed policy that
appears to conflict with it.
In case of contradiction between a proposal which is made the basis of the
contract and the terms of the policy document which is issued later, the
policy documents is likely to take precedence being the final and formal
expression of the agreement.

17(a) Define Contribution as an Insurance concept. (2 marks)


(b) Explain the operation of the doctrine of contribution at common law
showing vividly, the drawback(s) in the operation.
(2 marks)

(c) List and explain how Insurers have been able to change or override the
drawback(s) of the common law rule.
(26 marks)
Solution 17
(a) Contribution is the right of an insurer to call upon others similarly, but not
necessarily equally liable to share the cost of an indemnity payment.

(b) Contribution is governed by Common law. At common law, an insured


whose loss was covered by two or more policies cannot recover more than
an indemnity. He could claim against any of the insurers in any order. If
he chose to claim from one and he was paid in full, the paying insurer
would be saddled with the burden of bearing the cost of handling the
claim, pains of having to wait till the other insurers would pay and faced
with the possibility of dispute. That position of things was unacceptable to
Insurers and therefore devised ways round the Common law rule.

(c) These common law principles are frequently modified by:


i. Clauses in the Policy known as Contribution Conditions and
ii. Market agreements.

i. Contribution Conditions:
These are Clauses that set out how the loss is to be met if the insured
has another Policy which covers it. Examples are:

(a) Escape clauses


This is a condition that effectively forbids the insured from taking
our another policy without the consent of the insurers.

It will be stated that the policy will be avoided if the insured takes
out any further insurance on the same risk without notifying the
insurer and obtaining their consent.

It may also provide that the insurance will be invalid if the insured
already has a cover on the same risk with another insurer.

If the insured has two policies that cancel each other, the first in time
will prevail. See HOME INSURANCE COMPANY OF NEW
YORK V GAVEL.

An accidental overlap will not operate bring this condition into use.
See AUSTRALIAN AGRICULTURAL COMPANY V SAUNDERS.

(b) Other Non-contribution clause


This clause simply will state that there will be no liability for any
cover which is insured by another policy. The effect is to push the
whole of any such loss into the other insurer.

(c) More specific insurance clauses


A clause may be inserted in the policy that will state that where a
loss is covered by another more specific insurance, the Policy will
respond only when the cover provided by the more specific
insurance has been exhausted.

(d) Ratable Proportion Clauses: this is the most commonly used. It will
state that the insurers will be liable for rateable proportion only of
any loss that is also unknown by another policy.
ii. Markets Agreements. Insurers sometime agree amongst
themselves to modify the operation of Contribution. These
they do in two ways: They may agree to share losses in cases
where Contribution does not arise in law. Secondly, they
sometime agree to waive rights of subrogation where such
clearly exist, so that the whole of the loss is borne by one
insurer.

18 The content or terms of an Insurance contract may roughly be divided


into: Warranties and Conditions.

(a) Define these two terms (2 marks)


(b) How may warranties arise in an Insurance contract relationship?
(c) How may conditions be classified? (5 marks)
(d) Highlight the effects of breach of Warranties and Conditions. (2 marks)

Solution 18
(a)
(i) Warranties
The most important terms in an insurance contract which bring about the
most drastic effects if they are broken.
(ii) Conditions:
The terms other than warranties that imposes an obligation on the insured.

(b)
Warranties may be expressly stated in the policy.
Warranties may be implied into the policy document.
Warranties may be inferred from the basis of the contract clause.

(c)
Conditions precedent to contract
Conditions precedent to liability
Collateral conditions or (mere conditions)
Suspensive conditions or clauses describing the risk

(d)
i. Breach of warranty or condition
Cover terminates automatically and, in effect, the contract ends.
Cover terminates from the same time of the breach.
The insurers do not have to prove a connection between the breach and
any loss that has occurred.
The insurers cannot avoid a particular claim and allow the contract to
continue. They must either allow the contract to terminate, or waive (agree
to disregard) the breach.

ii. Breach of condition precedent to the contract


If a condition precedent is never fulfilled the contract is void ab initio
If the condition imposes a continuing obligation the effect is similar to a
breach of warranty, above, but cover may terminate only if there is a
casual connection between the breach and the loss.

iii. Breach of conditions precedent to liability


The insurers may avoid the particular claim
The policy as a whole is not avoided and remains in force.

iv Breach of collateral (or mere condition)


The insurers may not avoid the policy or the claim, but may claim
damages.

v. Breach of a Suspensive Condition or clause describing the risk


Cover is suspended for as long as the insured fails to comply with the
condition, but resumes if and when he starts to comply with it again.

D10

LOGO
CHARTERED INSURANCE INSTITUTE OF NIGERIA

DIPLOMA

OCTOBER 2013 EXAMINATION PAPER

SUBJECT D10
COMMERCIAL INSURANCE PRACTICE

Three hours are allowed for this paper

Fill in the information requested on the answer booklet and on form B.

Do not write on the question paper.

The answer booklet and this question paper must be handed in personally
by you to the invigilator before you leave the Examination Hall. Failure to
do this may result in your paper not being marked and you may be
prevented from entering for this examination in future.

D10
CHARTERED INSURANCE INSTITUTE OF NIGERIA

D10 COMMERCIAL INSURANCE PRACTICE

INSTRUCTIONS TO CANDIDATES

Three hours are allowed for this paper.


You should answer all questions in Part 1 and Two (2) out of the Four (4)
questions in Part II.

The paper carries a total of 200 marks distributed as follows:


Part I - 140 marks.
Part II - 60 marks.

You are advised to spend not more than two hours on Part I. You are strongly
advised to attempt ALL the required questions to score maximum possible
marks.

In attempting the questions, you may find it helpful in some places to make
rough notes in the answer booklet. If you do this, you must cross through these
notes before you hand in the booklet.

Answer each question on a new page. If a question has more than one part leave
several lines blank after each part.

D10
D10 COMMERCIAL INSURANCE PRACTICE
PART I
Answer ALL questions in Part I.
Each question carries 10 marks
Note form is acceptable where this conveys all the necessary information.

1. In a combined policy, there are many benefits that make it popular. State five
(5) of these benefits.

Solution 1
Benefits of combined policy include:
- there is only one Insurance policy, one premium, one renewal data, and one
document.
- a number of individual covers can be combined as required.
- The actual levels of cover and terms are tailored to meet the needs of the
individual business.
- They are more flexible than the standard package type of cover.
- It gives no room for arguments as to which section of the policy is cover or not.

2. When the schedule in a policy is not large enough to contain all the details of
the Property insured, there is another document that Insurer can use, what is
this document called and give the benefit of such device/method.

Solution 2
A Specification is prepared to contain details of the property insured when the
schedule is not large enough especially for more complex risks. The schedule may be
simplified to show for example, Refer to the specification attached. The benefits is
makes it possible for details of large risks that could be contained in the schedule to be
incorporated in the policy.

3. Write short notes on the followings:


(a) Dry Perils
(b) Wet Perils

Solution 3
(a) Dry Perils are exclusions under Assets protection Insurances and this include:
Explosion, Aircraft, Earthquake, Riot and Civil Commotion.

(b) Wet Perils are also exclusion which include storm, flood, bust pipes, and impact
damage.

4. What do you understand by Subsidence Cover?

Solution 4
Substance: this is the gradual movement or sinking of ground below a building causing
uneven settlement of the structure leading to cracking and damage to the building.

Applicable Exclusions are:


- Coastal or river erosion
- Defective Design
- Inadequate construction of foundation
- Movement is made up of ground
5. In a motor trade policy, list and explain briefly three (3) main element of this
policy.

Solution 5
(i) The Road Risks: This section covers those vehicles mainly on the road.

(ii) The Internal (Premises) Risk: This section covers the damage to the vehicle at
Insured premises.

(iii) The Asset business interruption, liability and other covers as insured under a
typical combined policy.

(iv) Details of driver, details of the vehicles, the use of the vehicle, area of use, length
of journey.

(v) - Under age Driver/an old driver


- Habit of the Insurer/Driver e.g.
A perpetual alcohol driving
- A less careful Driver/Insured as shown in the claim history
- One eyed or bad sighted Driver
- Length of experience of the Insured/Driver

6. In a normal Motor Insurance Cover, mention


(i) Three (3) physical hazards Underwriters always look for.
(ii). List three (3) moral hazards that can influence the judgment of a prudent
Underwriter when presented with Motor Insurance proposal.

Solution 6

a) They are:
(i) Third party insurance
(ii) Third party, fire and theft insurance
(iii) Comprehensive insurance
(iv) Contingent liability cover
(v) Restricted Road Traffic Act (RTA) cover only
b) The moral hazards are:
(i) the given of a wrong name by the proposer
(ii) the given of a wrong address by the proposer
(iii) the given of a wrong trade/business by the proposer

7. Immobilized plant recovery is an extension in Contractors all Risks


Insurance. What is its Usefulness?

Solution 7
It provides for the cost of recovering Mechanical plant such as earthmoving equipment
which can become bogged down in heavy ground condition even though the plant has
not been lost or damaged. Its usefulness is that it tend extend the CAR to this risk which
is not normally covered by the policy.

8. In selecting maximum indemnity period, there are some factors Insured will
need to consider, name four (4) of such factors.

Solution 8
The factors should include:

- The Time taken to carry our repairs


- The use of specialized machinery and the time taken to replace it
- Delay in replacing raw materials, components or parts
- Competition in the market and loyalty of customers
- Whether the business is seasonal.

9(a) The Employers Liability (Compulsory Insurance) Act 1969 exempted some
categories of employers. Mention these categories of employers.
(b) How would injury be defined in Employers Liability Insurance?

Solution 9
The categories of the exempted employers are

(a) - Nationalised Industries


- Police Authorities
- Local Government Authorities

(b) Bodily injury, death, illness or disease

10. There are various sources of information the Underwriter will consider in
Underwriting Liability Insurance. List any five (5).

Solution 10
These are:

- Proposal form
- Survey
- Confirmed claim experience
- Product brochures
- Sales literatures
- Internet site
- Through Governing Body of the said client or insured
11. What is the Role of Overriding Commission in Reinsurance Business?

Solution 11
Over-riding commission is to reimburse the ceding company (Insurer) for the expenses
she might have incurred in the process of securing the business(es) ceded to the Re-
Insurance Company.

12. State four (4) advantages of Reinsurance arrangement.

Solution 12
- Spreading of Risk
- Confidence Building
- Protection of business against risks that are catastrophe in nature
- Ability to acquire more risks
- It allows coding companies to acquire skills and knowledge from the
Reinsurance companies

13. There are conditions to be met before Insurers can be liable to pay a claim
arising from any bond issued to the Insured; Based on your experience, what
are these conditions?

Solution 13
- Bond holder fails to perform a specific duty.
- Bond holder fails to perform a duty properly.
- If the subject of the bond is dishonest

14. Business travel policy is becoming popular in the Nigerian Insurance Market.
What are the main Underwriting risks?

Solution 14
Underwriting risks include:
- The destination
- Period of time or duration of cover
- Age of the proposer
- Pre-existing medical conditions (if the trip is for health reason)
- The type of work to be undertaken (If the purpose of the visit is for paid manual
work)

Part II
Answer Two (2) out of the following Four (4) questions.
15. A building was insured against Fire and the contents against burglary. The
building was burgled and some contents were carted away. The reports of
the claims Adjuster show that the entrance door was not tampered with, but
the Burglars came through the ceiling of the house.

(a) ABZ Insurance Plc wants you to advise whether the claim is cover or
not. Give reason(s)?
(b) State the definition of theft as contained in Theft Act 1968.
(c) What are the three (3) Basic Fire Perils and mention seven (7) Allied
Perils.

Solution 15
(a) The claim is covered:

Though the entrance door was not tampered with according to the Adjusters
reports, but there was forcible and violent entry and theft did occur.

(b) Theft involving forcible and violent entry, auxiliary damage and hold-up

(c) The basic perils covered by the fire policy are:


Fire
Lightning
Domestic Explosion
The Allied perils are:
Explosion
Aircraft
Riot, civil commotion and malicious damage
Earthquake
Subterraneous/underground fire
Spontaneous fermentation
Storm/flood
Escape of water
Sprinkler leakage
Impact
Subsidence
Heave
landslip

16. Asset Protection Policy has become popular policy in the modern Insurance
Practice, this is due to the uniqueness of covers it provides:

(a) What do you understand by this policy?


(b) Mention three (3) common exceptions as it relates to Fire and special
perils.
(c) List four (4) additional clauses which may be added to an asset
protection policy.

Solution 16
(a) Asset Protection refers to the identification and insurance of all the
physical/material assets of a business. The most common of these are: the
buildings, machinery/plant and all other contents.

(b) Any three:


- Any consequential loss or damage
- Property more specifically insured
- Property insured by a marine policy
- Damage caused by pressure waves, terrorist damage, etc.

(c) Any four:


- Automatic reinstatement of the sum insured.
- Change of occupancy
- Construction and the heating of the building
- Foundation clause
- Non-invalidation clause
- Temporary removal clause

17(a) Loss of use has become one of the common extensions to Motor Insurance. As
an Insurance professional, what do you understand by loss of use and what
is the purpose of this extension in Motor Insurance Policy?

(b) Before a Motor Insurance policy can be fleet-rated by Insurance Company,


what is the minimum number of vehicles the Underwriter will consider?

(c) Some vehicles are regarded as special types. List four (4) of them.

Solution 17
a. Loss of use is an extension in motor insurance serving as an incentive to the
Insured person(s) to buy insurance from the insurer. Typically or generally, it is
exclusion in a standard commercial motor cover.

The purpose of loss of use is to extend the policy for the Insured by the Insurer to
cover some or all of the leasing or hire charges if the proposer is to have before
he is indemnified following a loss to the insured vehicle as a result of an
accidental damage, fire or theft.
b. Generally ten (10) vehicles are the minimum (but some insurers will reduce this
to (5) five).

c. Four from:
- Agricultural vehicles
- Forestry vehicles
- Buses and coaches
- Cranes
- Mobile shops
- Mobile plant
- Ambulances
- Bulldozers
- Hearses

18. What is the full meaning of the following?

(a)i. MPL
ii. PML
iii. NLE
iv. EPL

(b) What is the need for EML by Insurers?

(c) Differentiate between Proportional and Non-proportional treaty.

Solution 18
(a) MPL = Maximum Probable Loss
PML = Probable Maximum Loss
NLE = Normal Loss Expectancy
EPL = Estimated Probable Loss

(b) Insurers need EML to:


- Identify the maximum loss which is likely in normal circumstances.
- Assess and manage their exposure to risks.
- Ensure the underwriters are making use of the companys full capacity
- Protect the balance sheet
- Help in the purchase of their reinsurance programmes

(c) Proportional treaty (Reinsurance) is where an Insurer (the ceding company)


cedes a proportion of each risk. The reinsurer accepts the share of the risks
offered and will accept a proportionate share of premium and pays the same
proportion of the claim in the event of a loss.
Non proportional treat (Reinsurance) is based on the size of the expected loss
(an amount) rather than a percentage share of the risk. The ceding company only
cedes out a particular amount that is in excess of what she can bear. The loss for
this reason; it is usually referred to as excess of loss reinsurance as the loss has to
exceed a certain level (which may be Insurers Net retention) before a claim can
be made against the reinsurer.

D11

LOGO

CHARTERED INSURANCE INSTITUTE OF NIGERIA


DIPLOMA

OCTOBER 2013 EXAMINATION PAPER

SUBJECT D11
PERSONAL INSURANCE PRACTICE
Three hours are allowed for this paper

Fill in the information requested on the answer booklet and on form B.

Do not write on the question paper.

The answer booklet and this question paper must be handed in personally
by you to the invigilator before you leave the Examination Hall. Failure to
do this may result in your paper not being marked and you may be
prevented from entering for this examination in future.

D11
CHARTERED INSURANCE INSTITUTE OF NIGERIA

D11 PERSONAL INSURANCE PRACTICE

INSTRUCTIONS TO CANDIDATES

Three hours are allowed for this paper.

You should answer all questions in Part 1 and two (2) out of the four (4)
questions in Part II.

The paper carries a total of 200 marks distributed as follows:

Part I - 140 marks.


Part II - 60 marks.

You are advised to spend not more than two hours on Part I. You are strongly
advised to attempt ALL the required questions to gain maximum possible marks.
In attempting the questions, you may find it helpful in some places to make
rough notes in the answer booklet. If you do this, you must cross through these
notes before you hand in the booklet.

Answer each question on a new page. If a question has more than one part leave
several lines blank after each part.
D11
PART I
Answer ALL questions in Part I.
Each question carries 10 marks
Note form is acceptable where this conveys all the necessary information.

1. Using alternative methods of settlement, how can insurer choose to settle a


claim under the terms of the policy wordings?

Solution 1
An insurer can choose to settle claim in one of these four ways:

(a) payment of money (cash-in-lieu)


(b) Paying to repairer
(c) Replacement
(d) Reinstatement

2. Mention three (3) advantages of Arbitration as a means of dispute resolution


over litigation.

Solution 2
- It is cheaper than litigation
- It is faster than litigation
- Qualified arbitrators understand Insurance technical matters than the
court of law

3. Who is a loss Assessor? Differentiate a Loss Assessor from a Loss Adjuster.

Solution 3

Loss assessors are employed by the policyholders to assist them in the


preparation of their claim. The loss assessors will act on behalf of the
policyholder to negotiate the best claim settlement. They also recover
uninsured expenses.

The loss adjuster is an independent intermediary who function is to assist


in settling claims. Adjusters belong to CILA unlike the assessors. Assessors
represent the claimant while loss adjusters investigate claims on behalf of
insurers. Insurers pay the loss adjusters fees while the assessors collect
their fees from the insured.
4. What is ex-gratia payment? Why do you think that an Insurer should make
ex-gratia payment? Can an Insurer, having made ex-gratia payment, take
over the salvage?

Solution 4

Ex-gratia payment is the special payment insurers make when there is no


legal liability to do so. Insurers make ex-gratia payment to protect their
reputation and or for commercial purposes. Insurers can not take over the
salvage following a loss where the claim is settled on ex-gratia basis.

5. Mention and explain briefly the two (2) corollaries of Indemnity.

Solution 5

The two corollaries of indemnity are subrogation and contribution

Subrogation allows insurers to recoup any profit which the insured may
make from an insured event. It allows them to pursue any right or
remedies which the insured may possess which may reduce the loss.

Contribution provides that a policyholder can not recover more than


indemnity, even if they have several policies covering the same loss.

6. What are the factors limiting payment of Indemnity?

Solution 6

The factors limiting the payment of indemnity are:

(a) Sum insured: sum insured is the maximum recoverable under the
policy even where the true indemnity would be a much greater
figure

(b) Limit of Indemnity: this is the maximum payable by insurer. It is


commonly found in liability policies.

(c) Average: where there is under insurance, average reduces the


amount payable by insurer

(d) Excess; It is an amount that the insured bears when there is a claim,
no matter how small or large the claim
(e) Franchise: It is a fixed amount the insured pays when the loss has
not exceeded the amount of the franchise. The insurers pays the
whole claim when the loss exceeds the amount

(f) Deductible: A deductible is the name given to a large excess. The


insurer gives premium discount in return.

7. State the principle and the facts in Bufe vs Turner (1815).

Solution 7

The principle is utmost good faith. In Bufe V. Turner (1815) the insured
unintentionally did not disclose that there had been a previous fire in the
immediate vicinity of the insured property. The court rendered the policy
void.

8. What are the components of an Office Premium?

Solution 8

The components of an office premium are


(1) Pure Premium: The amount expected to cover the expected cost of
loss
(2) Contingency loading: It is the margin or reserve that is created to
cater for the loss which exceed the expected cost
(3) Expenses loading: The expenses of the Insurer is also considered
(4) Profit loading: there is a margin of profit that is loaded to the
premium

9. What is the definition of money in a travel policy?

Solution 9

Travel policy defines money as cash, bank or currency notes, cheques,


postal or money orders, travellers cheques, travel tickets, petrol or credit
vouchers, some insurers include credit cards and passports.
10. List five (5) powers of Financial Services Authority (FSA) when authorizing
and Monitoring Insurers.

Solution 10

Powers of financial services Authority are


(1) Authorisation
(2) Solvency
(3) Monitoring
(4) Intervention
(5) Conduct of business
(6) Winding up

11. What are the three features of a Survey Report?

Solution 11

Features of a survey report are:


(1) Full risk description
(2) Full risk assessment
(3) Recommendations for loss prevention
(4) A view in the adequacy of the sum insured

12. What is a product life? List and explain four (4) stages of this cycle.

Solution 12

A product life cycle is the stages in the sales history of a product. The
stages are (1) Introduction (2) Growth (3) Maturity (4) Decline

13. Explain Non-Proportional treaties.

Solution 13

Non-proportional treaties - this is an arrangement where the reinsurers


agree to pay an amount over and above (or in excess of) that which the
cedant agrees to pay. There are two types:

(a) Excess of Loss:


The reinsurers agree to pay any claim in excess of a particular
amount e.g. N2,000,000 in excess of N500,000

(b) Stop Loss:


It is not on claim per claim. It is on a particular account for a
particular period e.g. motor account of an underwriter for year 2013.
The intention is to prevent wild fluctuation in an insurers loss ratio.

14. Define the Principle that was established in Pawsey vs. Scottish union and
National (1908). What is the fact of Gaskarth vs. Law Union Insurance
Company (1876)?

Solution 14

The principle is that of proximate cause, which is defined as It is the


active and efficient cause which sets in motion a train of events that brings
about a result, without the intervention of any force stating and working
actively from a new and independent source.

In Garskarth V. Law Union Insurance Coy (1876) Fire damaged a wall and
left it weakened. Several days later a gale blew the wall down. It was
decided that fire was not the proximate cause of the wall falling down.

Part II
Answer TWO (2) out of the following FOUR (4) questions.

15(a) Define Contribution as a principle of Insurance.

(b) How can contribution arise?

(c) If a policy with Trustlink Insurance Plc has N5m sum Insured and the one
with Safelink Assurance Plc has N3m while the one with Legal and Liability
Insurance Plc has N2m.

If there is Contribution. How can these Insurers share a loss of N1.5m?

Solution 15

(a) Contribution is the right or an insurer to call upon others similarly,


but not necessarily equally liable to the same insured, to share the cost of
an indemnity payment.
(b) Contribution will only arise where the following conditions are met:

(1) Two or more policies of indemnity exist


(2) The policies cover a common interest
(3) The policies cover a common peril which gave rise to the loss
(4) The policies must cover a common subject matter
(5) Each policy must be liable for the loss
(6) There is a common interest

(c) Trustlink has sum insured of N5,000,000


Safelink has sum insured of N3,000,000
L & L Insurance has sum insured of N2,000,000

The formula to calculate Rateable Proportion is

Sum insured x loss


Total sum insured

Trustlink 5,000,000 x 1,500,000 = N750,000


10,000,000

Safelink 3,000,000 x 1,500,000 = N450,000


10,000,000

L & L Ins. 2,000,000 x 1,500,000 = N300,000


10,000,000

16. Write short notes on each of the following:

(a) Single article limit


(b) Radioactive contamination and explosive nuclear assemblies.
(c) Contractual liability
(d) Untraced Drivers Agreement
(e) Solvency Margin
(f) Implied duties

Solution 16

a. Single Article Limit: It means no one valueable item to be of greater


value than 5% of the contents sum insured. Item like gold, jewellery, silver
or other precious metals are usually included.
b. Radioactive Contamination and explosive nuclear assemblies: It is a
popular exclusion of a household policy. Damage caused by nuclear
weapon or related explosion is not covered by the policy. This damage has
the potential to be too big for individual insurer to carry. It is normally
placed in a pool i.e. market pools.

c. Contractual liability
This is a standard exclusion in all liability policies. It states that insurer will
not be involved in a claim arising out of the agreement they knew nothing
about and could not therefore have been taken into consideration when
assessing the premium which the insured has introduced through a
contract agreement.

d. Untraced Drivers Agreement


This agreement covers Hit and run cases where the negligent driver
remain unidentified. It applies to compensation for personal injury or
death and since 14/2/2003 to property damage.

MIB nominates one of its members to negotiate settlement on its behalf.


The cost being met by way of levies imposed by MIB (Motor Insurers
Bureau) members.

Guarantee fund section of MIB administers the operation.

e. Solvency margins
It is the amount by which asset exceed liabilities. Each Insurer is required
to maintain a minimum balance between its assets and its liabilities. The
NAICOM aims to minimize the risk of the insurance company having
insufficient funds to meet its present and future claim.

f. Implied Duties
These are unwritten duties which are imposed by common law. For
example the law requires that the insured:

- should act as though they are uninsured


- require to advise the appropriate authorities e.g. fire service in case
of fire
- must take every reasonable steps to extinguish fire
- must not hinder the insurer in the investigation of their claim
17. Detail ten (10) of the policy extensions that may be included in a private
motor policy.

Solution 17

The extensions under a private motor policy are


1. Damage to windscreens
2. Damage to or loss of rug, clothing and personal effect
3. Losses caused by young additional drivers
4. Loss of use
5. Personal accident benefits
6. Elections
7. Continental use
8. Transport by sea and Air
9. Racing, competitions, Rallies and trials
10. Breakdown cover
11. Courtesy cars

18(a) Give an historical background of travel Insurance.

(b) What are the popular extensions and exclusions in a travel policy?

Solution 18

a Travel Insurance was first provided for the benefit of businessmen on


overseas trips. Originally, cover was limited to personal accident cover
and loss or damage to baggage. The demand was increased and the cover
widened following worldwide expansion of air travel.

Most travel Insurance was originally written by tour operators now there
is a whole rage of suppliers including intermediaries, banks, motoring
organisations, sports and professional associations, as well as insurers
selling the products to the public.

b. Extensions to Travel Policy

(1) Hospital Cash benefit


(2) Delayed baggage
(3) Travel Interruption
(4) Travel delay
(5) Failure of tour organisers
(6) Lack of service or amenities
(7) Loss of passport
(8) Legal expenses

Exclusions
(1) Death, bodily injuries or sickness caused by drugs (unless medically
prescribed) or caused by intoxicants, insanity, venereal diseases,
pregnancy (some underwriters cover pregnancy up to seven months) or
an naturally occurring condition of gradual operating cause.

- Death and bodily injury consequent upon physical or mental defect


when taking hazardous pursuits like hockey, polo, etc.
- Suicide or self inflicted injury

(2) Loss of luggage


- By confiscation
- When the luggage was unattended
- Corneal caps, micro lenses, stamps, manuscripts camping equipment
- Cash/cheques unless reported to the police within 24 hours

(3) Damage to fragile articles


D12

LOGO

CHARTERED INSURANCE INSTITUTE OF NIGERIA

DIPLOMA

OCTOBER 2013 EXAMINATION PAPER

SUBJECT D12
LONG TERM BUSINESS

Three hours are allowed for this paper

Fill in the information requested on the answer booklet and on form B.

Do not write on the question paper.

The answer booklet and this question paper must be handed in personally
by you to the invigilator before you leave the Examination Hall. Failure to
do this may result in your paper not being marked and you may be
prevented from entering for this examination in future.
D12
CHARTERED INSURANCE INSTITUTE OF NIGERIA

D12 LONG TERM BUSINESS

INSTRUCTIONS TO CANDIDATES

Three hours are allowed for this paper. You should answer all questions in Part
I, and Two (2) out of the Four (4) questions in Part II

The paper carries a total of 200 marks distributed as follows:

Part I 14 compulsory questions 140 marks.


Part II 2 questions selected from 4 60 marks.

You are advised to spend not more than two hours on Part I. You are strongly
advised to attempt ALL the required questions to gain maximum possible marks.
Where a question is split into parts (a), (b) etc, marks for each part are only
shown if they are split unevenly between the parts and you should spend your
time in accordance with the allocation.

Answer each question on a new page. If a question has more than one part leave
several lines blank after each part.

It is important to show each step in any calculation, even if you have used a
calculator.

You may find it helpful in some places to make rough notes in the answer
booklet, if you do this, you should cross through these notes before you hand in
the booklet.

D12
PART I
Answer ALL questions in Part I.
Each question carries 10 marks.
Note form is acceptable where this conveys all the necessary information.

1. Carefully discuss why a life assured will always get less than the premium
paid in event of a Policy Surrender.
(10 marks)
Solution 1
- Breach of policy conditions
- No time to build adequate reserve
- Initial expenses or acquisition cost
- Investment opportunity denied
- Actual value depends or length of time the policy has been in force.

2. Identify ten (10) relevant information you can find in the contents of a typical
Medical Certificate of Cause of Death (MCCD).

Solution 2
1. Name of hospital
2. Hospital Number
3. Name of deceased
4. Age of the deceased
5. Date of death
6. Time of death
7. Year of death
8. Primary cause of death
9. Secondary cause of death
10. Name of Doctor
11. Qualification of doctor
12. Address of the hospital
13. Stamp of the hospital

3. Given that:
Lx = number of the start of age aged x
dx = number dying at age x
qx = the rate of mortality at age x
ex = the expectation of life age x

Also that a make table =


age Lx dx qx ex
x
0 100000 814 0.00814 73.413
40 96500 166 0.00172 35.349
41 96334 179 0.00186 34.409
42 96155 193 0.00201 33.473
45 95521 254 0.00266 30.684

What is the probability (Px) that a person aged 40 will survive for another one
year?

Solution 3
Px = Lx + 1 (5 marks)
Lx

Po = L41 = 96334
L40 96500 (3 marks)

= 0.99828 (2 marks)

4. Carefully enumerate ten (10) relevant information you can find in Know
Your Policyholder (KYP) form.

Solution 4
1. Name of Policyholder
2. Sex
3. Date of Birth
4. Marital Status
5. Address
6. Email
7. Occupation
8. Name of Insurance Company
9. Address of the insurer
10. Nationality of Policyholder
11. Means of identification
12. Name of agent/brokers
13. Agent/broker registration number
14. Source of fund

5. What important factor in your checklist will you be concerned about in the
event of a surrender of a mortgaged life assurance policy?
Solution 5
The counter signature of the mortgagee if the surrender was made by the
mortgagor. However, the signature verification is irrelevant if the request was
made by the mortgagee under the power of sale.

6. The law on assignment of life policies is contained in the Policies of


Assurance Act 1867. Discuss the obligations the Act imposed on Life
Insurance Companies.

Solution 6
a) Life offices must show appropriate office address where notice of
assignment can be served.

b) Life offices must acknowledge receipts of the notice of assignment.

c) Life offices must have a register for recording or notice of assignment.

d) They must put a caveat that such notice does not confer the assignee
ownership of the policy.

7. Describe the relevant information under the Family History of a typical


proposal form.

Solution 7
1. Age and state of health (if living), age at death, cause of death and year of
death of father, mother, sister(s) and brother(s).

2. Enquiry of named ailment suffered by any member of the proposers


family.

8. Identify various methods to deal with life assurance proposal with


substandard lives.

Solution 8
1. Grant limited cover
2. Exclude coverage of some risks
3. Charge extra premium
4. Rating-up by using an age higher than the proposers
5. Impose debt or lies on the policy
6. Postpone acceptance
7. Decline the risk present

9. A proposer for life assurance in your company disclosed some medical


conditions in her life in the proposal form. As an astute underwriter what are
those ten (10) medical conditions that will put you on enquiry?

Solution 9
1. Heart disease
2. Cancer
3. Obesity
4. Liver disease
5. Kidney disease
6. Diabetes
7. Mental disorder
8. HIV/Aids
9. Respiratory disease
10. Glandular disease
11. Eye disease
12. Disease of nervous system

10. The assessment of a proposal for life assurance is key. What underwriting
factors are considered during risk assessment?

Solution 10
1. Medical factors
2. Occupational factors
3. Residential factors
4. Financial factors

11. Discuss the contents of a life assurance claims or discharge voucher.

Solution 11
1. Policyholder name/claimant
2. Policy number
3. Policy type
4. Claims amount
5. Signature of claimant
6. Name of Insurer
7. Address of Insurer
8. Name of witness
9. Address of witness
10. Name of beneficiary

12. Mr. Okonkwo Okon Olajide has nine (9) children of which one is the only
male child (Two years old today). He approached you for a life assurance
policy on the life of the male child for his own benefit in demonstration of his
love for this child. Discuss if this man has legal right, with respect to insurable
interest, to do so.

Solution 12
This man does not have insurable interest, as parent, on the life of the child. This
is because parents do not suffer financial loss on the death of a child. If it were
possible, parents will be tempted to commit infanticide.

13. Carefully, enumerate the various types of investment products available in


the financial services sector.

Solution 13
1. Endowment policies
2. Shares
3. Unit Trust
4. Gilt-edged securities
5. Investment-linked insurance policies
6. Debentures
7. Government bonds
8. Company bonds
9. Insurance bonds
10. Options, futures, swaps etc

14. What is your understanding of money laundering? List the stages involved in
money laundering?

Solution 14
Money laundering is the process through which criminals and terrorist convert
money obtained illegally into legitimate funds.

The stages of money laundering are:


1. Placement
2. Layering
3. Integration

Part II
Answer TWO (2) out of following FOUR (4) questions.
Each question carries 30 marks

15. Discuss family Income Benefit policy.

Solution 15 (a)
Family Income Benefit provides for the payment of an Income. The policy pays
the selected level of income each year from the date of death of the life assured
until expiry of the policy. FIB policies are usually cheap because cover reduces
over the term of the policy.

(b)
An Insurance agent who assists an applicant to complete an application or
proposal from for Insurance shall be deemed to have done as the agent of the
applicant.

16. Your valuable assured wrote recently requesting for an alteration in the
policy. His wish is that the term of the policy be reduced from 20 years to 8
years. You observed from the file that the policy is in its 5 th year and is
encumbered (that is, there is a third party interest)
How will you treat this request?

Solution 16
1. Acknowledge the letter of request
2. In the letter, state that you will need the consent of the third party in
writing to effect the alteration.
3. Copy and send the letter to 3rd party
4. Wait for letter of discharge from the 3rd party
5. No alteration if the reply from the 3rd part is that the policy is still
encumbered
6. Alteration is effected if the 3rd party states that there was no need to
request his consent.
7. Also ensure that the reassure is informed for necessary adjustment.

17. A proposer for a term Assurance policy in your company is also requesting
for additional benefits to be added to the policy. Carefully state all additional
benefits that can be added to a basic life policy.

Solution 17
1. Waiver of premium
2. Disability benefit
3. Double accident benefit
4. Increasing cover option
5. Critical illness cover
6. Health care benefit

18. Discuss
(a) Operative Clause
(b) Life of another policy
Solution 18
(a) Operative Clause provides that the benefit stated in the schedule of the
policy will be paid provided that premium has been paid, the claim is
proved, policy ownership is proved and claims form (discharge voucher)
is completed.

(b) Life of another policy: In this policy is where a person effects a policy on
the life of another for his own benefit. The third party effecting the
insurance must have an insurable interest when the policy commences. For
example, a creditor effect a life policy on the life of a debtor and also
husband and wife (Married Woman Act).
D13
LOGO

CHARTERED INSURANCE INSTITUTE OF NIGERIA

DIPLOMA

OCTOBER 2013 EXAMINATION PAPER

SUBJECT D13
MARINE AND AVIATION INSURANCE

Three hours are allowed for this paper

Fill in the information requested on the answer booklet and on form B.

Do not write on the question paper.

The answer booklet and this question paper must be handed in personally
by you to the invigilator before you leave the Examination Hall. Failure to
do this may result in your paper not being marked and you may be
prevented from entering for this examination in future.
D13

CHARTERED INSURANCE INSTITUTE OF NIGERIA

D13 MARINE AND AVIATION INSURANCE

INSTRUCTIONS TO CANDIDATES

Three (3) hours are allowed for this paper which contains 20 compulsory
questions and carries a total of 200 marks.

Where a question is split into parts (a), (b) etc, marks for each part are only
shown if they are split unevenly between the parts and you should spend your
time in accordance with the allocation.

Answer each question on a new page. If a question has more than one part,
leave several lines blank after each part.

It is important to show each step in any calculation, even if you have used a
calculator.

You may find it helpful in some places to make rough notes in the answer
booklet, if you do this, you should cross through these notes before you hand in
the booklet.

D13- MARINE AND AVIATION INSURANCE


Each question carries 10 marks
Note form is acceptable where this conveys all the necessary information.

1. What do you understand by the terms Latitude and Longitude? Illustrate


your answer with diagrams.

Solution 1
Latitude is the angular distance of a place North or South of the Equator,
measured from the centre of the earth.

Diagram

Longitude is the angular distance of a place east or west of the Greenwich


Meridian (often called the Prime Meridian and accepted by international
agreement). The angle is measured at the centre of the Earth along the
Equator.

Diagram:

2. What is the essence of time difference? Give an example to buttress your


explanation.
Solution 2
Time difference is always important to appreciate and understand in all
forms of business, but especially when working with other major world
markets. Example:

1200hrs London time is


2100hrs Tokyo time and
0700 hrs New York time

3. State the relationship between Seaport, Airports and Insurance.

Solution 3
The situation of a seaport or airport has two main effects on insurance

a. The physical risk exposure. The size of a seaport its location, any weather
hazards or congestion problems may produce additional hazards to the
Marine Insurer. Also some airports are located in mountainous areas
which Insurers may consider these more of a physical risk.

b. Its trade or passenger level - a seaport handling non-hazardous, low value


raw material would not offer insurers major concern. But a very busy
seaport regularly handling shipment of oil or liquid natural gas would.

4. What are:
(a) Customs Invoice
(b) Certified Invoice

Solution 4
a. Customs Invoice: is used by the relevant authorities to check that no duty
liability is being avoided by inaccurate pricing.

b. Certified invoice: is a commercial invoice bearing a detailed statement as


to value and origin and signed by the exporter. In some countries, this type
of invoice is required to be legalized by the buying nations consulate in
the UK before the goods can enter the country.

5. What is the legal definition of Bill of Exchange?

Solution 5
A bill of exchange is legally defined as an unconditional order in writing,
addressed by one person to another, signed by the person giving it,
requiring the person to which it is addressed to pay on demand or at a
fixed or determinable future time, a sum certain in money to, or to the
order of a specified person or to bearer.

6. Enumerate the services of a Freight Forwarder.

Solution 6
Freight forwarder arranges the dispatch of export products to particular
countries to which the goods are consigned. They arrange transport,
prepare some of the necessary documents and also arrange insurance
cover, if agreed by the exporter. They also arrange groupage service,
whereby small consignments may be grouped with those of other
exporters.

7(a) Define Excess of Loss Ratio

(b) What other name is given to this type of Reinsurance Cover

Solution 7
a. Excess of Loss Ratio is a form of protection which enables the reassured to
reduce their net liability on individual risks to a figure within their
compass. The loss ratio of the original insurer is stopped at an agreed
percentage, if the loss ratio in any particular year exceeds that agreed
figure, the reinsurer will pay the excess amount. The reinsurer will not
give unlimited cover restricting liability to an agreed percentage of the net
retained lines of the original insurer.

b. Excess of Loss ratio also known as Stop Loss

8. Define
(a) Quota share Treaty Reinsurance

(b) What do you understand by Advance Freight?

Solution 8
a Quota Share Reinsurance: is a reinsurance agreement whereby the
reassured cedes a predetermined proportion of all business for specified
part thereof to their reinsurer.
b. The Continental law allows pro rata distance freight i.e. if the shipowner,
through no fault of his own, is unable to complete the voyage due to one
reason or the other, is entitled to the proportion of freight earned before
the adventure was terminated. English law insists that freight is payable
only on right delivery of goods, unless otherwise agreed by both parties.

9. State the Insurance principles involved in the cases of


(a) North of England Iron SS. Ins Assn v. Armstrong (1870)
(b) Goole & Hull Steam Towing Co. v. Ocean Marine Ins. Co. (1928)

Solution 9
a. The Insurance principle of claims settlement in respect of Total
loss/Recovery from the negligent third party by the Insurer.

b. Insurance Principle of Claims Settlement in respect of Partial


loss/Recovery from the negligent third party by the Insurer.

10. What are the provisions of S.78 of the MIA 1906 Duty of Assured (Sue and
Labour).

Solution 10
A shipowner who incurs expenditure in trying to save their vessel when
there is no other property involved, but is unsuccessful, and the vessel
sinks, will recover their expenditure, subject to the provision regarding
under-insurance and the limitation of the amount insured.

11. Highlight the Exclusions under the Radioactive Contamination Exclusion


Clause.

Solution 11
The exclusions under the Radioactive Contamination Exclusion Clause are:
Losses, liability or expenses directly or indirectly caused by

a. Ionizing radiation from or contamination by radioactivity from any nuclear


fuel or from any nuclear waste or combustion of nuclear fuel.

b. Radioactive, toxic, explosive or other hazardous, or contaminating properties


of any nuclear installation, reactor, or other nuclear assembly or component
c. Any weapon or war employing atomic or nuclear fusion and/or fusion or
other like reaction or radioactive force or matter.

12. A vessel was forced to berth at a port in Alepo Syria, whilst at berth there was
an explosion from a missile fired by the opposition forces. The vessel with all
its Cargo was completely bowed down. Can the Cargo owner claim in respect
of his lost cargo?

Solution 12
The situation in Syria is a war situation, clearly excluded from any liability
under the Marine Insurance Act, war clauses exclusions. This stipulate that
in no case shall the Insurance cover loss, damage or expenses caused by
war, civil war, revolution, insurrection or civil strife arising from any
hostile act by or against a belligerent power.

13. Is partial interest of any nature insurable in Marine Insurance? Explain.

Solution 13
A partial interest of any nature is insurable in Marine Insurance, for
example a bulk item in respect of two separate contracts can be shipped,
with the two contractors having insurable interest in their individual part
of the shipment. Each individual interest will be for his specific share of
the shipment.

14. The Marine Insurance Act 1906 Section 18 specified when some facts need not
be disclosed. Specified these circumstances.

Solution 14
Under section 18 of the MIA 1906 the following facts need not be disclosed

a. Any circumstance which diminished the risk


b. Any circumstance which is known or presumed to be known to the insurer
i.e. matters of common knowledge which insurers in the ordinary cause of
their business ought to know.
c. Any circumstance as to which information is waived by the insurer.
d. Any circumstance which it is superfluous to disclose by reason of any
express or implied warranty.
15(a) Mention the dual task of an Aviation Insurance broker.
(b) What is the benefit in placing a business with the Lloyds Broker?

Solution 15
a. The Dual tasks are:

i) Obtaining the best possible terms with acceptable security for their client
ii) Providing underwriters with a service by issuing adequate documentation
and ensuring punctual payment of premium.

The broker is obliged to keep underwriters fully informed of events concerning a


risk and must reveal all pertinent details.

b. The benefit of placing business with Lloyds Broker, is that the


underwriters are able to subscribe to business in almost every country in
the world. Lloyds also provides a worldwide network of agencies for
purpose of loss adjustment, legal advice, statistical information and
general commercial intelligence.

16. There are some specified Rules under the Marine Insurance Act 1906. State
the first two rules i.e.
(a) Rule 1: Lost or not lost
(b) Rule 2: From

Solution 16
These Rules as specified by the Act are for the construction of a policy.

a. Rule 1 Lost of nor loss is used where the subject matter is insured lost or
not lost and the loss has occurred before the contract is concluded, the
risk attached unless at such time the assured was aware of the loss and the
insurer was not.

b. Rule 2: from where the subject matter is insured from a particular


place, the risk does not attach until the ship starts on the voyage insured.

17. One of the objectives of the Aviation Industry in respect of international


regulations is to allow an operator to limit liability for death or bodily injury
to a passenger or loss/damage to baggage or cargo. Mention these laws as
regards the regulation.
(a) Japanese Initiative - 1992
(b) Australian Legislation - 1994/1995

Solution 17
a. Japanese Initiative 1992 Japan requires its carriers to waive limits and
in particular for amounts up to SDR 100,000 to waive defence of Article 20
of the Warsaw Convention.

b. Australian Legislation 1994/1995


Australia requires its carriers to increase limits to SDR 260,000

18(a) Highlight the Sub Sections of the Section III legal liability to Passengers of
the London Aircraft Insurance Policy.
(b) Under Exclusions applicable, mention the specific exclusions.

Solution 18
a. The three sub sections of the Section III are:

i. Coverage
ii. Exclusion applicable
iii. Limits of Indemnity

b(i) Employee liability


(ii) Operational Crew

19. State the Cover granted under the Additions and Deletions (Applicable to
Hull only) (AVN 17A) Policy.

Solution 19
The cover granted under the Additions and Deletions (Applicable to Hulls Only)
(AVN 17A) are:

a. Automatic extension of the policy to include at pro rata additional


premium in respect of further Aircraft added during the currency of the
policy provided such Aircrafts are owned or operated by the insured and
are of same type, and value as the ones already covered.
b. Inclusion of additional Aircraft of other types or different values shall be
subject to special agreement and rating by insurers prior to attachments.

c. Aircraft which have been sold or disposed of shall be deleted from this
policy and the insured shall be entitled to pro rata return of premium
provided no claim has arisen and become payable under this policy in
respect of such Aircraft, and that this policy is not cancelled by virtue of
such deletion.

20. What are the basic provisions of Section 2 Extortion and Hi-jack Expenses of
the Appendix 2 Airport Owners and Operators Liability policy.

Solution 20

The basic provision of section two: extortion and Hi-jack expenses of the
Appendix 2 of the Airport Owners and operators liability policy are:

1. Indemnify the Assured subject to the terms condition etc and limitations
set out below and up to the limit stated in the schedule for 90% of any
payment property made in respect of:
a. threats against any Aircraft stated in the schedule or its passengers or crew
made during the currency of the policy.
b. Extra expenses necessarily incurred following confiscation, hi-jacking of
any Aircraft stated in the schedule.

2. No cover will be provided under this section of the policy in any territory
where such insurance is not lawful and the assured is at all times
responsible for ensuring that no arrangements of any kind are made which
are not permitted by the proper authorities.

D17

LOGO

CHARTERED INSURANCE INSTITUTE OF NIGERIA

DIPLOMA
OCTOBER 2013 EXAMINATION PAPER

SUBJECT D17
OPERATIONAL MANAGEMENT

Three hours are allowed for this paper

Fill in the information requested on the answer booklet and on form B.

Do not write on the question paper.

The answer booklet and this question paper must be handed in personally
by you to the invigilator before you leave the Examination Hall. Failure to
do this may result in your paper not being marked and you may be
prevented from entering for this examination in future.

D17
CHARTERED INSURANCE INSTITUTE OF NIGERIA

D17 OPERATIONAL MANAGEMENT

INSTRUCTIONS TO CANDIDATES

Three hours are allowed for this paper.


You should answer all questions in Part 1 and Two (2) out of the Four (4)
questions in Part II

The paper carries a total of 200 marks distributed as follows:


Part I - 140 marks
Part II - 60 marks

You are advised to spend not more than two hours on Part I. You are strongly
advised to attempt ALL the required questions to score maximum possible
marks.

In attempting the questions, you may find it helpful in some places to make
rough notes in the answer booklet. If you do this, you must cross through these
notes before you hand in the booklet.

Answer each question on a new page. If a question has more than one part leave
several lines blank after each part.

D17
PART I
Answer ALL questions in Part I.
Each question carries 10 marks.
Note form is acceptable where this conveys all the necessary information.

1. Differentiate between a Manager and a Leader.

Solution 1
A manager is a person who performs the specific functions of management and
who holds on official title or plays on official part within the organization
while

A leader is a person who is able to influence others to pursue goals. As a


results leadership does not require the trappings of officialdom.

2. Identify the eight (8) main views of leadership.

Solution 2
(i) The existence of special leadership traits
(ii) Behavioural theories
(iii) Situational leadership
(iv) Action Centred leadership
(v) Transactional leadership
(vi) Transformational leadership
(vii) Intrapreneurship
(viii) Facilitative leadership

3. State three (3) advantages of delegation

Solution 3
(i) Managers gain more time to concentrate on high-level activities such
as strategic planning.

(ii) Subordinates are able to take on greater responsibility and learn new
skills in self-contained and more managerial branches.

(iii) Delegation encourages team spirit by allowing staff to participate


more in the organizations tasks and decision.

4. List the six (6) styles of decision making identified by T.Y Awoba in late
1970s.

Solution 4
(i) No thought
(ii) Compliant
(iii) Logical
(iv) Emotional
(v) Intuitive
(vi) Hesitant
5. Enumerate five (5) types of interviews normally conducted.

Solution 5
(i) Termination Interview
(ii) Grievance Interview
(iii) Disciplinary Interview
(iv) Career Development Interview
(v) Appraisal Interview

6. State five (5) means of reducing the effects of stress.

Solution 6
(i) Providing break areas for staffs to get away and release tension
(ii) Providing work variety to reduce monotony
(iii) Providing a quality and conducive working environment
(iv) Ensuring team members have clear and achievable goals, objectives,
targets.
(v) Giving praise and positive feedback for successes
(vi) Avoiding a blame approach when someone is unsuccessful
(vii) Mediating between team and management requests, if necessary,
negotiate.
(viii) Keeping team disruption to a minimum by dealing with conflicts
immediately
(ix) Involving the team in decision making and ensuring they accept
responsibility for their actions.
(x) Ensuring there are clear procedures for handling stress related
problems

7. What are the attributes of a successful project?

Solution 7
(i) Stated objectives and deliverable are achieved
(ii) Delivered on time
(iii) Within budget
(iv) Perform to an agreed specification
(v) Meet pre-defined success criteria
(vi) Satisfy the sponsor and beneficiaries

8. Differentiate between Management Control and Operational Control.

Solution 8
(i) Operational control relates to individual tasks while management
control is concerned with the sum effect of individual tasks.

(ii) Many operational control decisions can be automated or


programmed. In contrast, management control, relying on
judgement and qualitative as well as quantitative factors, is always
non-programmed.

9. What are the seven (7) steps to Success in motivation?

Solution 9
- Support the team by sharing knowledge
- Understand the peoples needs and limitation
- Collate and analyse data
- Communicate clearly and effectively
- Encourage enthusiasm and ideas from the team
- Set Specific targets for individuals and the team
- Support colleagues in achieving team goals

10. Identify the four (4) phases of employee performance appraisal.

Solution 10
Performance Appraisal Phases
(i) Assessing the individual
(ii) Communication of the Assessment
(iii) Identifying training and development needs
(iv) Setting objectives

11. State five (5) sources organizations use to help them find appropriate staff for
recruitment.

Solution 11
(i) Graduate recruitment
(ii) Advertising
(iii) Recruitment Agencies
(iv) Personal recommendations
(v) Internal promotion

12. What are the points to consider in designing a Tele-Interview?

Solution 12
(i) Agree the competencies and personal characteristics required
(ii) Decide the way they are to be assessed
(iii) Decide the scoring process
(iv) Develop a script and from to collect the information
(v) Train the tele-Interviewers
(vi) Agree on the success criteria

13. What is Performance Management?

Solution 13
Performance management is a systematic approach to the management of
people using performance and development as forces to motivate
individual employees to contribute to organizational goals while at the
same time realizing their own career plans.

14. Define Counseling and list the key steps in counseling.

Solutions 14
a. Counseling can be defined as creating opportunities for an individual to
explore and discover ways of achieving their potential as a human being.

b. The key counseling steps are:

(i) Identify what the real problem is


(ii) Establish trust and moving forward with empathy
(iii) Considering possible actions that could help to improve the
situation
(iv) Agreeing the action to be taken

Part II
Answer TWO (2) out of the following FOUR (4) questions.
Each question carries 30 marks.

15. At corporate level, planning needs to cover the key areas that will allow the
organisations objectives to be achieved. What are these areas?

Solution 15
Key areas in planning
(i) Setting objectives
(ii) Identifying what needs to be done for those objectives to be achieved
(iii) Creating the most appropriate organization structure
(i) Agreeing and establishing a consistent management style
(ii) Agreeing and setting budgets
(iii) Agreeing staff incentives
(iv) Setting sales targets
(v) Planning the most efficient use of material resources
(vi) Setting timetables and deadlines
(vii) Identifying contingency plans

16. The foundation of staffing and recruiting is to have a sound and logical
recruitment and selection process, the aim of which is to find people who will
deliver high quality performance. It is essential that organisations test for key
skills, attitudes and attributes at the right stage in the recruitment procedure.
Identify the stages of the recruitment procedure.

Solution 16
Stages of the Recruitment Procedure
(i) Determine the Business needs
(ii) Job description, competencies, person specification
(iii) Decide recruitment sources
(iv) Prescreening
(v) Assessment of Application Form and/or CVs.
(vi) Decide interview approach and structure
(vii) Interview and Selection process
(viii) Selection Decision
(ix) Offer and Confirmation
(x) Evaluation

17(a) Identify the two approaches known as Coaching Spectrum

(b) With the aid of a table, differentiate between these two (2) approaches.

Solution 17
a) They are:
- Management Coaching
- Achievement Coaching
b)
Management Coaching Achievement Coaching
A process by which a A continuous and
manager, through discussion participative process
and guided activity, helps a whereby the manager or 18(a) What
do member of staff to solve a coach provides both the you
problem or carry out a task opportunity and the understand by
better. The focus is on encouragement for an
practical improvement of individual to address
performance and the their needs effectively in
development of specific skills the context of personal
and organization
objectives
Coach does most of the Coach does most of the
Talking Listening
Putting in pulling out
- Skills - Potential
- Knowledge - Commitment
- Experience - Expertise
Benchmarking?

(b) Explain the types of benchmarks.

Solution 18

a. Benchmarking is a process that allows a company to compare its own


progress with that of a comprehensive standard. It usually means the
establishment of performance measures that enable a company to analyse
its efficiency against competitors or leading companies in the Industry.

bi. Internal Benchmark: These compare the performances of divisions and


departments within the same organization.

ii. External Benchmark: These contrast the companys overall performance


with competing firms e.g. Profitability, rate of return on capital employed,
growth etc.
iii. Functional Benchmark: This covers an assessment of the companys main
functions and processes and compares them against the same functions
and processes in other organisations but not necessarily competitors.
D18
LOGO

CHARTERED INSURANCE INSTITUTE OF NIGERIA

DIPLOMA

OCTOBER 2013 EXAMINATION PAPER

SUBJECT D18
PROPERTY CLAIMS HANDLING

Three hours are allowed for this paper.

Fill in the information requested on the answer booklet and on form B.

Do not write on the question paper.

The answer booklet and this question paper must be handed in personally
by you to the invigilator before you leave the Examination Hall. Failure to
do this may result in your paper not being marked and you may be
prevented from entering for this examination in future.
D18

CHARTERED INSURANCE INSTITUTE OF NIGERIA

D18 PROPERTY CLAIMS HANDLING


INSTRUCTIONS TO CANDIDATES

Three (3) hours are allowed for this paper which contains 20 compulsory
questions and carries a total of 200 marks.

Where a question is split into parts (a), (b) etc, marks for each part are only
shown if they are split unevenly between the parts and you should spend your
time in accordance with the allocation.

Answer each question on a new page. If a question has more than one part,
leave several lines blank after each part.

It is important to show each step in any calculation, even if you have used a
calculator.

You may find it helpful in some places to make rough notes in the answer
booklet, if you do this, you should cross through these notes before you hand in
the booklet.
D18
D18- PROPERTY CLAIMS HANDLING

Each question carries 10 marks.


Note form is acceptable where this conveys all the necessary information.

1. Give examples of how tortuous liability may arise.

Solution 1
a. Fire as a result of the negligent act of a contractor

b. Water damage following inadequate or incurred plumbing

c. Impact by road vehicle

d. Subsidence can result from nuisance

2. State Day One Average Condition in Property Policies.

Solution 2
This means, if at the time of damage, the Declared Value of the property insured
is less than the cost of Reinstatement at the inception of the policy, then the
liability of the insurer shall not exceed the proportion of the Declared Value to
the cost of Reinstatement.

3. Highlight the Insurers Rights following a claim

Solution 3
- Right to take possession
- Right to keep possession
- Right to salvage
- Right of Entry

4. Write notes on Animal Act 1971

Solution 4
a. - There is a strict liability for damage by trespassing livestock Defences
- Contributory negligence
- Escape wholly due to the fault of the person suffering the loss

b. There is also liability for damage by dangerous species

5. The property insurance market is made up of many players. List them.

Solution 5
- Policyholder
- Brokers and Agents i.e. Intermediaries
- Insurers
- Reinsurers
- Third party Claim administrators
- Assessors
- Trade and Market associations
- Superintendent
- Actuariant

6. Discuss the issues involved in the case of Carter V. Boehm (1766)

Solution 6
- Carter was the Governor of a fort in Sumalia (modern day Indonesia)
- He effected a policy with Lloyds Underwriters (Boehm) against the fort being
captured by foreign enemies
- The fort was captured less than a year later
- Boehm denied liability on the basis of non disclosure
- Carter argued that the facts were common knowledge.

7. Define policy warranty.

Solution 7
A warranty is an undertaking by the policyholder that some specified thing shall
or shall not be done

Or

That some condition shall be fulfilled or


Whether the policyholder denies or affirms the existence of a specified fact
or facts.

8. The term All Risks is a misnomer because no policy covers all risks. The All
Risks Policy has many exceptions which have been for ease of reference
broken down to groups. List these exceptions.

Solution 8
All risks policy do not cover all conceivable risk. Below are some of the
exclusions
- Exclusion of events
- Exclusion of property
- Standard war exclusion
- Standard ionizing radiation exclusion
- Northern Ireland Overriding clause
- Consequential loss

9. You are to deliver a paper on Impact Damage Extension during the next
meeting of the Fire Offices Committee. Outline the contents of your paper.

Solution 9
This is a paper presentation and should be handled as such the follow must be
the substance of the paper
(a) - Cover under this extension may be restricted to impact damage by any
road vehicle, train or animal.
- There must be physical impact i.e. Collision

(b) Exclusions
- It does not cover damage by insects, birds or domestic pets
- It does not cover damaged by the imposed load of a vehicle rolling over
- It does not cover damage by loads falling from vehicles

10. Give five examples of misrepresentation.

Solution 10
- Non-disclosure of a criminal record not spent
- Non-disclosure of previous claims
- Non-disclosure of an insurers refusal to accept or renew a policy
- Misrepresentation of security devices
- Misrepresentation of the proposers business
- Misrepresentation of the construction of the premises

11(a) Define Explosion.


(b) To what extent is Explosion covered in the Standard Fire Policy.

Solution 11
a. Explosion has been defined as

i. An event that is violent, noisy and caused by a very rapid chemical or


nuclear reaction or
ii. The bursting out of gas or vapour under pressure

b. It only covers explosion of domestic boilers and/or gas used for domestic
purposes or heating and/or lighting.

12. Write all that you know of Stock Debris Removal.

Solution 12
- Stock Debris Removal is not covered under the standard fire policy
- It may be covered by extension
- The cost incurred can be substantial particularly if the debris is
contaminated and has to be disposed at licensed sites.
- The cost should not be paid unless there is stock debris removal cover.

13. The judgement in the case of North British and Mercantile Insurance
Company V. London, Liverpool and Globe Insurance Company (1877) made
an important contribution to insurance practice.
(a) What is this contribution?
(b) Give a summary of this case

Solution 13
a. Contribution between property policies does not arise where different
persons insure in respect of different rights.

b. This case is also known as King and Queen Granaries case.


The granary was destroyed along with its contents including grain owned
by Radocanachi and Co who effected a fire cover on the grain.

The granary owner also had a fire policy on customers grain as a Bailee.

The two insurers entered into this friendly case to test contribution.

It was held that the granary insurer was liable with no contribution as the
policies cover different interests.

14. Write notes on


(a) Aggregate deductible
(b) Definitions in Property Policies
(c) Rebuilding guarantee

Solution 14
- Aggregate deductible
A deductible is the first part of a claim that the insured bears.

An aggregate deductible relates to the total of all such claims in the year.

- Definitions in Property Policies


Insurers incorporate definitions into property policies to give a clear
picture of the key terms used in the policy.

Where this is not done words will be used in their ordinary dictionary
meaning.

- Rebuilding guarantee
Thus may be found in policies covering buildings.

By this clause, Insurers give an undertaking that irrespective of the cost


they will pay the reasonable cost of reinstating the building.

15. Explain the independent liability method of loss apportionment.

Solution 15
- It requires a calculation to be made in respect of each of the policies to
establish the sum that the policy would have paid if it were the only policy
in force.

- It takes into account all adjustments that would be made to the claim
under the policy.

- The total of the independent liabilities of all the policies is then


calculated.

- The loss is apportioned in the ratio that the independent liability of each
policy bears to the total independent liabilities.

16. Describe the mechanism typically causing wall finishes to fail.

Solution 16
- The most common cause of defect is frost damage.

- If render is not maintained and if airline cracks which can develop in the
normal cause of clients are not filled and decorated, what can penetrate
and when it freezes it will make the cracks bigger until eventually the
render becomes detached, or water is allowed to penetrate the building.

17. Before alleging fraud, what degree of proof is required by the Insurer?

Solution 17
- In civil cases, liability is determined on the balance of probabilities.
- In criminal matters, guilt must be established beyond reasonable doubt
- Allegations of fraud against the policy holder is a serious allegation
though a civil matter.
- The degree of proof required is higher than that applicable to civil
matters but not as high as those relating to criminal matters.

18. Warranties are commonly found in property policies. Discuss a typical Alarm
Warranty.

Solution 18
- The specification for the alarm must have been agreed with insurers.
- It must be maintained in full working order
- The premises should not be left unattended unless the alarm has been set
- Keys must be removed when the alarm is set
- Security of alarm codes must be maintained

19. List the factors you would consider in calculating your companys overall
claims reserve.

Solution 19
- Aggregate of all the case by case reserves plus.
- Allowance for IBNR
Plus
- Indirect claims handling expenses

20. Discuss the judgment in the case of Ryland V. Fletcher (1868).

Solution 20
- It is a case of strict liability
- The judgment established as follows:
a) If a person for his own purposes brings on his land and collects and
keeps their anything likely to do mischief if it escapes, keeps it at his
peril.

b) If he does not do so he is prima facie an answerable for all the


damage which is the natural consequences of its escape.

Admissible Defences
- The escape was due to the plaintiffs default
- The escape was the consequences of vis major
- It was an act of God

Das könnte Ihnen auch gefallen